Wrong Answers Flashcards

You may prefer our related Brainscape-certified flashcards:
1
Q

The defendant’s neighbor owned an authentic major league baseball signed by Babe Ruth. The defendant asked if he could show it to some friends who were visiting. The neighbor agreed as long as he kept it in the display case, which the defendant promised to do. In fact, the defendant intended to use the ball in a pickup game. During the game, the ball was hit over the fence and into a yard with a guard dog, which had chewed up several other balls that had previously landed in the yard. The dog did the same to that ball. When the neighbor learned what happened to the ball, he pressed charges against the defendant.

If the defendant is convicted, he will most likely be found guilty of what crime?

A

The defendant is guilty of larceny by trick because he obtained possession of the baseball by means of a misrepresentation.

How well did you know this?
1
Not at all
2
3
4
5
Perfectly
2
Q

A student and a few of his friends were making their way to spring break. Along the way, the old van that they were driving broke down. Not wanting to miss any part of spring break festivities, the student asked the mechanic on duty at the repair shop for a rush job. The mechanic provided the student with a repair estimate, and the student, on the basis of the estimate, authorized the repair and promised to pay when he came back to pick up the van. When the mechanic called the student to tell him that the van was repaired, the student, rather than paying for the repair, told one of his friends that the mechanic had agreed to finance the repair charges and that the only thing left to do was pick up the van in the garage’s parking lot. The student handed the friend a key to the van and told him to go pick the van up so that they could continue their trip to spring break. The friend did so.

The mechanic makes a criminal complaint against the student for larceny of the van. If the case is prosecuted, will the student likely be found guilty?

A

The student will most likely be found guilty. Larceny is the taking and carrying away of the personal property of “another” with the intent to permanently deprive the other person of the property. It is possible to commit larceny of your own property if another person, such as a bailee, has a superior right to possession of the property at that time. Because the mechanic had a right to possession of the van until he was paid, the student committed larceny when he had his friend take the van without the mechanic’s consent.

How well did you know this?
1
Not at all
2
3
4
5
Perfectly
3
Q

An art restorer, after attending art school for a number of years, secured a job restoring the paintings for an art museum. After several years on the job, the artist discovered that he could imitate the artwork of nearly any artist. He decided that he could make some extra money copying the artwork of up-and-coming artists, while staying away from more well-known artists to reduce his chance of getting caught.

An art collector searching for a painting by a new artist saw the restorer at an art fair selling various paintings, one of which appeared to be by the new artist. The restorer was selling the painting for $100. The collector thought that the price was very low and that the painting should probably sell for around $500, but she bought the painting anyway, giving the restorer $100 after the restorer confirmed that the painting was an original from the new artist. After taking it to an art appraiser for insurance purposes, she discovered that the painting was a forgery. However, she also discovered that the painting’s frame was worth about $125.

With which theft offense may the art restorer be charged?

A

False Pretenses because he conveyed title to the $100 dollars

How well did you know this?
1
Not at all
2
3
4
5
Perfectly
4
Q

A weather vane collector placed the following ad in a newspaper: “I will pay $300 for information as to where I can purchase a Connecticut copper weather vane with Victorian Serpentine Motif in good condition.” The ad, which included contact information, was placed on Friday, December 5, and was to run a full week, starting on Sunday the 7th. A friend of the collector’s knew of his quest for the weather vane, but she did not see the newspaper ad. On Sunday morning, she saw at a swap meet the exact type of weather vane the collector sought and called him to tell him about it. The collector hurried down to the swap meet and got his weather vane at a good price.

On Monday, December 8, the friend saw the collector’s ad in an open newspaper. She called the collector and asked him to pay her $300 for the tip. After he declined, saying the offer was revoked when he purchased the weather vane, she sued the collector for her reward.

Will she prevail?

A

The friend cannot recover because she did not know of the offer. Although the collector’s placement of the ad was an offer, his friend did not accept by giving the requested information, because she did not know about the ad when she gave the collector the lead on the weather vane.

How well did you know this?
1
Not at all
2
3
4
5
Perfectly
5
Q

General Rule on Public Offers

A

The general rule on public offers is this: an offer of reward is an offer to enter into a unilateral contract, and if made to the public generally, it may be accepted by anyone to whom it becomes known. One who performs the requested act has done all that is necessary for acceptance, but if he does not intend that his acts constitute an acceptance, no contract results.

How well did you know this?
1
Not at all
2
3
4
5
Perfectly
6
Q

A microbrewery and a farming operation entered into a written contract for the sale of barley. The microbrewery agreed to buy from the farm “all the barley that the microbrewery requires” in its manufacture of beer, for a period of five years, at a mutually agreed-upon price. Under the contract, the microbrewery would place its orders on the first of each month and the barley would be delivered within five business days.

For the first two years of the contract, the microbrewery placed its orders on the first of each month for either four or five barrels of barley. At the beginning of the third year of the contract, an article about the microbrewery appeared in a national newspaper, causing its popularity to soar. The following month, the microbrewery placed an order for 20 barrels of barley. The farm could not meet the increased demand and refused to deliver the 20 barrels. The microbrewery sued the farm for breach of contract.

Will the microbrewery be successful in its suit?

A

The farm will prevail. Under UCC section 2-306(1), quantities subject to requirements contracts may not be unreasonably disproportionate to any stated estimate or, in the absence of any stated estimate, to any normal or otherwise comparable prior requirements

How well did you know this?
1
Not at all
2
3
4
5
Perfectly
7
Q

Question
A large producer of bread wrote to a distributor of flour, asking, “How much will you charge to supply my needs for flour for the next year?” The distributor replied in writing that it could supply the producer with all the flour it would need next year at a specified price per pound. The producer wrote back, “Your offer to supply me with flour is hereby accepted, provided that you agree to a 10% discount if payment is made within 10 days from date of billing.”

What should the producer’s reply concerning a 10% discount be characterized as?

A

A rejection. The producer’s reply is a conditional acceptance, which is a rejection of the offer. This question deals with the “battle of the forms” provision of the UCC. Under section 2-207 of the UCC, an acceptance containing additional or different terms is effective unless the offeree expressly makes his acceptance conditional on assent by the offeror to the additional terms. When an acceptance is made expressly conditional on the acceptance of new terms, it is a rejection of the offer. The conditional acceptance is essentially a new offer, and the original offeror may form a contract by expressly assenting to the new terms.

How well did you know this?
1
Not at all
2
3
4
5
Perfectly
8
Q

An owner of a piece of waterfront property contracted in writing with a contractor to rebuild the owner’s dock in accordance with plans and specifications prepared by the owner. The agreed contract price was $50,000, $25,000 of which was payable on May 1 when the job was to commence and the balance due upon completion of the work.

On March 1, the contractor notified the property owner that the contractor would lose money on the job at that price, and would not proceed with the work unless the property owner agreed to increase the price to $80,000. The property owner did not respond to the contractor, instead making a written contract with a third party to repair the dock, commencing May 1, for $60,000, which was the fair market cost of the work to be done. On May 1, both the contractor and the third party showed up at the dock to begin work, the contractor telling the property owner that he had decided to take the loss and would repair the dock for $50,000 as originally agreed. The property owner dismissed the contractor and allowed the third party to begin work on the dock.

In a contract action by the contractor against the property owner, is the contractor likely to prevail?

A

The property owner will prevail because the contractor breached. When the contractor notified the property owner on March 1 that he would not perform his obligations on a binding contract unless he was given more money, he committed an anticipatory breach. That breach gave the property owner the right both to terminate the contract and to engage a new contractor to complete the work.

How well did you know this?
1
Not at all
2
3
4
5
Perfectly
9
Q

A manufacturer and a buyer entered into a written contract for the manufacturer to produce and sell to the buyer 2,000 widgets at a price of $20 per widget. The contract expressly provided that the buyer shall have no liability under the contract unless 2,000 widgets are delivered to the buyer at his place of business no later than July 1. On July 1, 1,800 widgets meeting the buyer’s specifications were tendered by the manufacturer. The remaining 200 widgets were tendered on July 5. The buyer refused to accept any of the widgets.

In an action by the manufacturer against the buyer, which of the following would best support the manufacturer’s case, assuming it can be proven?

A

Delivery of the 200 widgets on July 1 was delayed by a storm which disrupted the shipper’s activities, and was not the manufacturer’s fault. The storm may have made delivery on time impossible, which may excuse performance under the doctrine of impracticability of performance. This defense can be used not only to excuse performance totally, but also to excuse the delay in full performance. Impracticability discharges the duty to perform to the extent of the impracticability; so if the storm rises to the level of impracticability and prevented the delivery of the 200 widgets, the manufacturer’s duty to perform with respect to those widgets would be discharged. In other words, failure to make perfect tender would be excused to the extent of the short delivery.

How well did you know this?
1
Not at all
2
3
4
5
Perfectly
10
Q

A shopkeeper loaned a long-time employee $1,500 from his personal bank account because a family illness was causing the employee unexpected financial difficulties. Because the employee had proved himself to be trustworthy, there was no writing evidencing the loan and no payback date established; it was understood that the employee would repay the loan when he was able to do so. Sometime later, the shopkeeper’s nephew asked him if he could help fund a business that he was starting up. Because most of the shopkeeper’s assets were currently tied up, he asked his employee if he would be in a position to repay the $1,500 loan. The employee promised to repay the loan on the following Monday, so the shopkeeper told the employee to pay the $1,500 directly to his nephew. Immediately thereafter, the shopkeeper informed the nephew to expect $1,500 from the employee on the following Monday. When Monday came, the employee decided he would rather tender the money to the shopkeeper than to someone he did not know, and the shopkeeper accepted the money.

If the nephew never receives any money from the shopkeeper, will he succeed in an action against the employee for the $1,500?

A

The shopkeeper validly assigned his right to receive the money to his nephew. However, this assignment was revocable, and it was revoked when the shopkeeper accepted the money from the employee. A creditor’s right to receive money due from a debtor is a right that can be assigned, regardless of whether the debt is evidenced by a writing. By telling the employee to pay the money to the nephew, the shopkeeper manifested an intent to transfer his rights completely and immediately to the nephew. Neither a writing nor consideration was required for this assignment to be valid. However, these factors do not affect revocability. This assignment was not given for value. Such a gratuitous assignment is generally revocable. An exception to this rule arises when the assignor is estopped from revoking because he should reasonably foresee that the assignee will change his position in reliance on the assignment and such detrimental reliance occurs. Here, there is no indication that the nephew in fact changed his position detrimentally in reliance on the assignment. Consequently, the general rule of revocability of a gratuitous assignment applies.

How well did you know this?
1
Not at all
2
3
4
5
Perfectly
11
Q

A landscaper entered into a written contract with a developer to landscape a 30-house subdivision at a price of $4,000 for each house. The contract provided for payment of the $120,000 only on completion of the landscaping for all the houses. After completing 20 houses, the landscaper demanded payment of $80,000. The developer refused. The landscaper then walked off the job without doing any landscaping on the other 10 houses. The developer refuses to pay the landscaper.

If the landscaper sues the developer, what damages should the court award the landscaper?

A

The landscaper may recover $80,000 less the developer’s damages resulting from the breach. A contract is divisible if it is possible to apportion the parties’ performances into corresponding pairs. Here the landscaper’s and developer’s performances can be apportioned into corresponding pairs: for each house landscaped by the landscaper, there is a corresponding payment of $4,000 owed by the developer. The contract itself states the price as $4,000 per house, rather than $120,000 for the entire job. If a party performs some of the units of a divisible contract, he is entitled to the agreed-on price for those units even if he fails to perform the other units.

How well did you know this?
1
Not at all
2
3
4
5
Perfectly
12
Q

A high-volume pleasure-boat retailer entered into a written contract to sell a customer a power boat for $120,000. The retailer could obtain from the manufacturer, for $90,500, as many of these boats as it could sell. As the contract provided, the customer paid the retailer $40,000 in advance and promised to pay the full balance on delivery of the boat. The contract contained no provision for liquidated damages. Prior to the agreed delivery date, the customer notified the retailer that he would be financially unable to conclude the purchase; the retailer thereafter resold the boat that the customer had ordered to a third person for $120,000 cash.

If the customer sues the retailer for restitution of the $40,000 advance payment, which of the following should the court decide?

A

The customer should recover $40,000 minus the retailer’s lost profit. The correct measure of damages is the lost profits of the retailer. That amount should be deducted from the deposit and the balance returned.

How well did you know this?
1
Not at all
2
3
4
5
Perfectly
13
Q

A citizen of State A filed a breach of contract action against a citizen of State B in a State A state trial court. The State B defendant timely and properly removed the action to the United States District Court for the District of State A. The defendant then filed a motion to dismiss the action based on insufficient service of process. Following a hearing, the court found that service was proper and denied the motion. The defendant then filed her answer, responding to the merits of the complaint and asserting that the case should be dismissed on the grounds that another action was pending between the same parties for the same cause in a State B state court. The State A Rules of Civil Procedure provide that a party waives the right to seek dismissal on that ground if the party files a pre-answer motion to dismiss and does not assert that ground in the motion.

Should the federal court hold that the defendant has waived the right to seek dismissal based on the pendency of the same cause in another court?

A

The defendant has not waived this defense. Unlike the state rule here, Federal Rule of Civil Procedure 12 does not require the defendant to raise the defense that another action is pending between the parties in the first responsive pleading. While the federal court exercising diversity of citizenship subject matter jurisdiction must apply state substantive law (here, State A law), applicable Federal Rules of Civil Procedure supersede state law in federal court as long as the rule comports with the requirements of the Rules Enabling Act (i.e., the rule governs practice and procedure and does not modify or abridge substantive rights)

How well did you know this?
1
Not at all
2
3
4
5
Perfectly
14
Q

A car buyer, a resident of State A, wished to purchase a new car. After shopping both online and in person for the best price, the buyer decided to purchase a car from a dealer in State B. The buyer drove to the State B dealership, signed a sales contract, paid for the car, and drove the car back home to State A. Three months later, the buyer was in an accident in State A while driving the new car. The other driver in the accident was a resident of State B who was just passing through State A. The other driver filed a negligence action against the buyer in a court in State B.

Does the buyer have sufficient contacts with State B such that a State B court could exercise personal jurisdiction over the buyer for the negligence action?

A
How well did you know this?
1
Not at all
2
3
4
5
Perfectly
15
Q

To satisfy a debt owed to a creditor, a son executed and delivered to the creditor a warranty deed to a large tract of undeveloped land. The creditor promptly recorded the deed. Shortly thereafter, she built a house on the property and has lived there ever since. The son never actually owned the land. It belonged to his father, but the father had promised to leave the property to the son.

Later, the father died and his will devised the property to the son. Pressed for money, the son then sold the land to an investor by warranty deed, which the investor promptly recorded. Although the investor paid full value for the property, he purchased it strictly for investment and never visited the site. He therefore did not realize that the creditor was living there, and knew nothing of the son’s earlier deed to the creditor.

The jurisdiction in which the land is located has the following statute: “A conveyance of an estate in land (other than a lease for less than one year) shall not be valid against any subsequent purchaser for value without notice thereof unless the conveyance is recorded.”

Which of the following is the most likely outcome of a quiet title action brought by the creditor against the investor?

A

The creditor prevails, because the investor was not a purchaser for value without notice of the creditor’s interest. Purchaser had inquiry notice.

How well did you know this?
1
Not at all
2
3
4
5
Perfectly
16
Q

Must a junior mortgagee be named as a party to a senior mortgagee’s foreclosure action?

A

Yes, a junior mortgagee must be named as a party to a senior mortgagee’s foreclosure action because it has the right to pay off the senior mortgage to avoid being wiped out by foreclosure. Foreclosure destroys interests ( e.g., liens, mortgages, leases, easements) junior to the mortgage being foreclosed. Thus, if a senior mortgage is in default, a junior mortgagee has the right to pay it off (i.e., redeem it) to avoid being wiped out by its foreclosure.

How well did you know this?
1
Not at all
2
3
4
5
Perfectly
17
Q

A landowner in fee simple signed a promissory note for $10,000 to a bank, and secured the note by a mortgage of her land to the bank. The mortgage was duly recorded. The landowner then sold the property to an attorney, who assumed and agreed to pay the mortgage to the bank on the land. The attorney did not make payments on the mortgage note to the bank. The bank, following appropriate statutory procedures, foreclosed the mortgage and gave notice to both the landowner and the attorney that it intended to sue for any deficiency. At the foreclosure sale, the property sold for $6,000. The bank now sues both the landowner and the attorney for $5,000, which is the remaining amount of the unpaid principal and interest on the note plus costs of foreclosure.

Against which party will the bank be successful in obtaining a judgment?

A
  • Both the landowner and the attorney. They are jointly liable.
How well did you know this?
1
Not at all
2
3
4
5
Perfectly
18
Q

An owner obtained a loan of $60,000 from a bank in exchange for a promissory note secured by a mortgage on his land, which the bank promptly and properly recorded. A few months later, the owner obtained another loan of $60,000 from a lender, in exchange for a promissory note secured by a mortgage on the land, which the lender promptly and properly recorded. Subsequently, the owner sold the land to a buyer for $150,000 and conveyed a warranty deed. The buyer expressly agreed with the owner to assume both mortgages, with the consent of the bank and the lender. A few years later, the bank loaned the buyer an additional $50,000 in exchange for an increase in the interest rate and principal amount of its mortgage on the land. At that time, the balance on the original loan from the bank was $50,000. Shortly thereafter, the buyer stopped making payments on both mortgages and disappeared. After proper notice to all appropriate parties, the bank instituted a foreclosure action on its mortgage, and purchased the property at the foreclosure sale. At that time the principal balance on the lender’s mortgage loan was $50,000. After fees and expenses, the proceeds from the foreclosure sale totaled $80,000.

Assuming that the jurisdiction permits deficiency judgments, which of the following statements is most accurate?

A

The bank’s original mortgage has priority in the proceeds, followed by the lender’s mortgage, and only the lender can proceed against the owner because the bank modified its mortgage after the owner had transferred to the buyer. If the landowner enters into a modification agreement with the senior mortgagee, raising its interest rate or otherwise making the agreement more burdensome, the junior mortgage will be given priority over the modification. On the other hand, the owner will not be liable to pay off the balance of the bank’s loan, because when a mortgagee and an assuming grantee subsequently modify the original obligation, the original mortgagor is completely discharged of liability. The owner had nothing to do with the modification agreed to by the bank and the buyer that increased the amount of the mortgage debt, and will not be even secondarily liable for that amount.

How well did you know this?
1
Not at all
2
3
4
5
Perfectly
19
Q

Question
A father who was paralyzed in an accident and unable to work refused to accept financial support from relatives or the government to help him care for his family. When one of his children became seriously ill and needed medical attention, the father refused to allow the child to receive medical care at the local public health clinic because of his feelings regarding the acceptance of charity. The father thought that the child was starting to get better, but then she took a turn for the worse and died from her illness.

What crime has the father most likely committed?

A

Involuntary Manslaughter. A father has a duty to provide the necessities for his child. His failure to do so has caused the child’s death. Such an omission is criminal.

How well did you know this?
1
Not at all
2
3
4
5
Perfectly
20
Q

A man asked a coworker who was a wine collector to lend him a bottle of expensive wine to put in his liquor cabinet, because he was inviting his mother over for dinner and wanted to impress her. The coworker permitted the man to take a bottle of wine worth $700 to his apartment for the dinner as long as he returned it the next morning. As he had planned all along, the man instead invited his girlfriend over for a romantic dinner, at which they drank the bottle of wine. The next day the man told his coworker that he had been mugged on his way home and that the muggers made off with the wine. Suspicious, the coworker found the empty wine bottle in the man’s trash from his apartment.

If the man is charged with theft in a common-law jurisdiction, of which theft offense is he most likely to be convicted?

A

Larceny by Trick. The man can be convicted of larceny by trick. The owner of the wine gave possession of the bottle of wine to the man, but clearly did not give up ownership, because possession was transferred on the coworker’s explicit promise to return it the next day. Therefore, a theft crime of some sort was committed, but that crime is not pure common law larceny because there was no trespassory taking. If the man obtained possession of the wine by fraud or misrepresentation, then he is guilty of larceny by trick. Because it apparently was the man’s intent to drink the wine all along, the possession was obtained by fraud and the crime is larceny by trick, not embezzlement (the crime which would have occurred if he obtained rightful possession).

How well did you know this?
1
Not at all
2
3
4
5
Perfectly
21
Q

Question
A felon planned to break into the rental storage unit next to his that contained valuable electronic equipment. He went to a hardware store to purchase a crowbar. The proprietor sold him the crowbar even though he told her that he needed it to break into someone’s storage unit. After the purchase, the felon went to the storage facility with his friend. The felon told the friend that he had lost the key to his storage unit and did not have time to contact the facility’s manager, so they needed to break into the unit to get his equipment. Because the felon had a bad back, the friend pried open the door with the crowbar and carried the equipment out to the car. A silent alarm was triggered and the pair were apprehended shortly after leaving the facility.

Can the proprietor and the friend be convicted as accomplices to larceny?

A

Neither the proprietor nor the friend would be convicted as accomplices to larceny. Mere knowledge that a crime may result from the aid provided is insufficient for accomplice liability so that is why the proprietor is not guilty.

How well did you know this?
1
Not at all
2
3
4
5
Perfectly
22
Q

A homeowner decided to destroy his home by fire in order to collect the insurance. A neighbor’s house was located a short distance from the homeowner’s home. The homeowner knew that there was a strong wind blowing towards the neighbor’s home; while he did not want to burn the neighbor’s home, he nevertheless set fire to his own home. The fire department was unable to save the homeowner’s house. They did manage to put out the fire moments before it spread to the neighbor’s home, which suffered damage from smoke and soot. The jurisdiction’s arson statute covers burning one’s own dwelling as well as the dwelling of another, but is otherwise unchanged from the common law.

If the homeowner is charged with attempted arson of the neighbor’s home, is he likely to be found guilty?

A

No, because he did not intend to burn the neighbor’s house. The homeowner will be found not guilty because he did not have the requisite mental state. To convict a person for an attempted crime, the prosecution must establish that the defendant had an actual specific intent to cause the harm prohibited by the statute and committed an act beyond mere preparation in furtherance of that intent. Those elements-specific intent and act-are required regardless of the mental state required by the target offense.

How well did you know this?
1
Not at all
2
3
4
5
Perfectly
23
Q

A drug smuggler had just returned home after smuggling in a large quantity of cocaine in the false bottom of his suitcase. As he was about to leave his house again to deliver the cocaine to his contact in the city, a police officer arrived with a trained drug-sniffing dog and asked him if he could come in and ask him some questions. The smuggler declined but the officer stepped into the doorway, and the dog immediately caught the scent of the cocaine and pulled the officer toward the suitcase in the hallway. Based on the dog’s clear indication that the suitcase contained narcotics, the police officer opened the suitcase and found the cocaine. The smuggler was then arrested and the cocaine and suitcase seized.

At a pretrial hearing, should the judge grant the smuggler’s motion to suppress evidence of the cocaine in the suitcase?

A

Yes, because the search and seizure required a warrant. Don’t pick the answer that is too broad

How well did you know this?
1
Not at all
2
3
4
5
Perfectly
24
Q

A suspect was arrested on a charge of bank robbery. After formal charges had been filed, the suspect was scheduled for a lineup identification procedure. The suspect’s attorney was notified of the lineup and arrived at the station prior to the scheduled lineup. He was directed to wait in the hallway outside the lineup room. When the suspect was escorted into the lineup room, the officer acknowledged the attorney but did not motion for him to follow them into the lineup room. The attorney remained seated in the hallway. Inside the lineup room, the six members of the lineup, including the suspect, stood on one side of the one-way mirror in full view of the witnesses gathered on the other side in the viewing area. The lineup members stepped forward one by one for a closer inspection. After the first two lineup members had been presented, an officer realized that the suspect’s attorney was not present and immediately escorted him to the viewing area. The attorney arrived in time to witness the presentation of the suspect and the remaining three members of the lineup. Two witnesses then identified the suspect as the robber.

At trial, if the defense objects to the introduction of the lineup evidence, should the objection be granted?

A

Yes, because the right to have counsel present at a post-charge lineup includes the right to have counsel present for the entire lineup. The objection should be granted. A post-charge lineup is a critical stage of the prosecution at which a defendant has the right to counsel. Once the government has initiated adversary judicial criminal proceedings, the presence of counsel is a prerequisite to the conduct of a lineup. This right attaches as soon as the accused is within sight of a potential identification witness.

How well did you know this?
1
Not at all
2
3
4
5
Perfectly
25
Q

As part of a deal to raise the federal debt limit, Congress passed a statute by a greater than two-thirds vote in both houses giving the President authority to cancel particular spending provisions that are contained within legislation that he signs into law. The statute provided that Congress could override the President’s decisions only by a three-fourths vote. As soon as the statute went into effect, a Senator who had voted against the statute filed suit in federal district court, challenging its constitutionality.

Is the Senator likely to succeed in her lawsuit?

A

The Senator will not succeed because she lacks standing to challenge the statute. The Supreme Court has held that members of Congress lack standing to challenge a law authorizing the President to exercise a line item veto (such as the statute here), reasoning that the injury is not concrete and personal, but rather is institutional in that it is shared by all members of Congress.

How well did you know this?
1
Not at all
2
3
4
5
Perfectly
26
Q

Congress adopted a law granting a state university $2.5 million to study the properties of snowflakes. An outraged taxpayer filed suit to challenge the appropriation.

Which of the following is the best argument against this challenge?

A

The best argument is that the taxpayer lacks standing. Generally, taxpayers lack standing to challenge federal appropriations. There is an exception for expenditures violating the Establishment Clause, but that is not the case here.

How well did you know this?
1
Not at all
2
3
4
5
Perfectly
27
Q

A woman whose children attended a charter school learned that the children of the woman’s neighbor who attended a parochial school received a hot lunch paid for, in part, through federal expenditures enacted under Congress’s spending power. The charter school received no funding from the federal government and was not allowed to participate in the hot lunch program. The woman challenged this federal expenditure as a violation of the Establishment Clause.

For her to bring the suit, at the very least what must the woman allege?

A She pays federal income taxes and has children who are not eligible for the lunch program because they attend the charter school.She pays federal income taxes and has children who are not eligible for the lunch program because they attend the charter school. - no response given

B She pays federal income taxes and the use of federal funds in this manner is an improper taxing and spending method.She pays federal income taxes and the use of federal funds in this manner is an improper taxing and spending method. - not selected, this is the correct answer

C She pays federal income taxes and has children who are of school age.She pays federal income taxes and has children who are of school age. - no response given

D She pays federal income taxes and has children who are of school age, and the use of federal funds in this manner is an improper taxing and spending method.

A

The woman must allege that she pays federal income taxes and that the use of federal funds in this manner is improper under Congress’s taxing and spending power. In general, a taxpayer has no standing to challenge the expenditure of federal funds. The major exception to this rule is where the taxpayer alleges that the expenditure was enacted under Congress’s taxing and spending power and exceeds some specific limitation on that power, in particular the Establishment Clause. Typically, neutral benefit programs that provide aid to private schools do not violate the Establishment Clause if they treat religious schools the same as other eligible private schools. This program, however, may be problematic because it provides a benefit to religious schools that isn’t available to other schools. Thus, the woman would have standing to contest this federal expenditure. The woman does not have to have children to bring this challenge, let alone children in the charter school. However, she would have to be a taxpayer.

How well did you know this?
1
Not at all
2
3
4
5
Perfectly
28
Q

In compliance with a federal statute that permits government agencies to sell or give away surplus government property, the Secretary of State directed that one of the State Department’s surplus airplanes be given to a church. The Secretary knew that the church planned to use the plane to fly medical supplies to its missions in Third World countries. These missions provide medical assistance, but they also attempt to evangelize residents of the countries in question, and the Secretary was aware that, in addition to medical supplies, the plane might transport Bibles and religious tracts translated into local languages. Had the Secretary not ordered the plane to be given to the church, it would have been sold at a very reasonable cost to a nonprofit organization that helps teach young people the fundamentals of piloting and maintaining aircraft.

Which of the following parties would be most likely to have standing to sue to prevent the Secretary of State from making the gift to the church?

A

A member of the nonprofit flying association is most likely to have standing to challenge the gift. To have standing to challenge government action on constitutional grounds, a person must show that he has a concrete stake in the outcome of the litigation. This is to ensure adequate presentation of the issues. To have such a stake, the potential litigant must show that he has an injury in fact caused by the government that is more than the theoretical injury that all persons suffer when the government engages in unconstitutional acts, and that a decision in his favor will eliminate his harm. A member of the flying association can show both components here: If the gift is unconstitutional, the association has suffered more than a theoretical injury-it has lost the opportunity to purchase the airplane from the federal government at a good price, and a decision in the club’s favor will eliminate the injury because it will then be able to purchase the plane. Thus, the member of the nonprofit flying organization has standing.

How well did you know this?
1
Not at all
2
3
4
5
Perfectly
29
Q

While driving in a city in State A, a truck driver and a motorcycle rider simultaneously struck a pedestrian. The pedestrian is a citizen of State A, whereas the driver and rider are citizens of State B. The pedestrian properly sued both drivers in a federal district court in State A, seeking $100,000 based on negligence.

If the truck driver wants to file a claim for damages to their truck totaling $7,500 against the motorcycle rider, may they do so?

Responses

A No, because the amount in controversy is too small.No, because the amount in controversy is too small. - no response given

B No, because the truck driver and motorcycle rider are both citizens of State B.No, because the truck driver and motorcycle rider are both citizens of State B. - no response given

C Yes, the truck driver must assert the claim or they will be foreclosed from asserting it later.Yes, the truck driver must assert the claim or they will be foreclosed from asserting it later. - no response given

D Yes, the truck driver may assert the claim against the motorcycle rider, but they are not required to do so.

A

The truck driver may assert the claim, but they are not required to do so. As a general rule, a party may assert a crossclaim against a co-party only if the crossclaim arises from the same transaction or occurrence as the original action or a counterclaim. Furthermore, such claims are permissive, in that the crossclaim will not be barred at the conclusion of the pending litigation. Here, the truck driver’s claim for damages to their truck would relate to the pedestrian’s negligence claim and is thus a proper crossclaim. There is supplemental jurisdiction over the claim because it arises from a common nucleus of operative fact as the pedestrian’s claim, and the withdrawal provisions contained in the supplemental jurisdiction do not apply because they apply, if at all, only to claims by a plaintiff.

How well did you know this?
1
Not at all
2
3
4
5
Perfectly
30
Q

On December 6, the owner of an electronics store sent a written request to a computer manufacturer asking for the price of a certain laptop computer. The manufacturer sent a written reply with a catalog listing the prices and descriptions of all of his available computers. The letter stated that the terms of sale were cash within 30 days of delivery. On December 14, by return letter, the store owner ordered the computer, enclosing a check for $4,000, the listed price. Immediately on receipt of the order and check, the manufacturer informed the store owner that there had been a pricing mistake in the catalog, which should have quoted the price as $4,300 for that computer. The store owner refused to pay the additional $300, arguing that his order of December 14 in which the $4,000 check was enclosed was a proper acceptance of the manufacturer’s offer.

In a suit for damages, will the manufacturer prevail?

Responses

A Yes, because his first communication stated terms calling for cash within 30 days of delivery.

B Yes, because of the mistake as to price.

C Yes, because his first communication did not constitute an offer.

D No, because the store owner’s December 14 letter was a proper acceptance of the manufacturer’s offer.

A

The store owner’s December 14 letter was an acceptance. Whether the letter was an acceptance depends on whether the manufacturer’s letter was an offer, because an acceptance is a manifestation of assent to an offer. For a communication to be an offer, it must create a reasonable expectation in the offeree that the offeror is willing to enter into a contract on the basis of the offered terms. There must be a promise, undertaking, or commitment to enter into a contract with certain and definite terms. Courts usually hold that if a statement is made broadly, such as in an advertisement or catalog, it will not constitute an offer because it is not reasonable to expect that the sender intended to make offers to all who received the advertisement; rather, the courts usually find such advertisements to be invitations seeking offers. However, price quotations may be considered as offers if given in response to a specific inquiry. The courts will look to the surrounding circumstances, and here a court would probably determine that the catalog that the manufacturer sent was an offer because it was sent in response to the store owner’s specific inquiries about prices on a specific computer and it included delivery terms and conditions of sale.

How well did you know this?
1
Not at all
2
3
4
5
Perfectly
31
Q

A builder contracted to build a house for a newly married couple. Terms of the contract provided that the builder would receive the contract price when the building was fully completed. Just when the builder had completed one-half of the structure, a tornado struck the area and demolished the building.

What is the builder entitled to recover from the couple under the contract?

Responses

A Nothing.

B One-half of the contract price.

C One-half of the fair market value of what remains of the house.

D Cost of materials and reasonable labor costs.

A

The builder will not be able to recover anything from the couple under the contract because he has not performed his duty. Under the parties’ contract, the builder’s completion of the house was a condition precedent to the couple’s duty to pay. The condition precedent was not discharged by the destruction of the work in progress because construction has not been made impossible, but rather merely more costly—the builder can rebuild. Thus, he is not entitled to any recovery. Note, however, that a number of courts will excuse timely performance because the destruction was not the builder’s fault.

How well did you know this?
1
Not at all
2
3
4
5
Perfectly
32
Q

A woman sued her ex-husband for installments due under their divorce property settlement agreement. The ex-husband defended the suit on the ground that the ex-wife was in breach of the agreement. However, the court granted judgment for the ex-wife. Now, the ex-wife is suing her ex-husband for other installments due under the agreement. The ex-husband raises the defense that the agreement was void and illegal because of fraud perpetrated by his ex-wife at the time the agreement was signed. The ex-wife moves to strike her ex-husband’s defense, claiming issue preclusion.

Should the court grant the motion?

Responses

A No, because the issue was never litigated.

B No, because judgment on one installment does not bar subsequent actions on other installments.

C Yes, because the husband should have raised the defense in the first action.

D Yes, because the issue of fraud is never waived.

A
How well did you know this?
1
Not at all
2
3
4
5
Perfectly
33
Q

A department store buyer and a manufacturer of food processors entered into a written contract whereby the manufacturer would sell to the buyer 50 of its top-of-the-line models for $100 each. When the delivery arrived on May 15, several days early, the buyer noticed that the food processors were a different model that did not have all of the features as the top-of-the-line model that was ordered. The buyer contacted the manufacturer and told him that he was rejecting the food processors that were delivered to him and expected the manufacturer to send 50 top-of-the-line models immediately. The manufacturer replied that because of a backlog of orders that had not yet been filled, the top-of-the-line models could not be delivered until August 15. Because the department store had contracted with a restaurant to deliver three top-of-the-line models by May 31, the buyer delivered three of the nonconforming food processors along with a promise to replace them with three top-of-the-line models in mid-August. The buyer returned the remaining food processors to the manufacturer.

How much could the department store recover from the manufacturer for the three food processors that it delivered to the restaurant?

A

The department store was entitled to recover contract damages from the manufacturer for the three food processors that it accepted. If the buyer accepts goods that breach one of the seller’s warranties, the basic measure of damages is the difference between the value of the goods as delivered and the value they would have had if they had been according to the contract, which is the difference between the processors and the market price.

How well did you know this?
1
Not at all
2
3
4
5
Perfectly
34
Q

On February 1, the owner of a bowling alley read in a magazine an ad from a major manufacturer of bowling balls offering sets of 40 balls in various weights and drilled in various sizes for $10 per ball. The owner immediately filled out the order form included in the ad for the 40 balls and deposited it, properly stamped and addressed, into the mail. On February 2, the bowling alley owner received in the mail a letter from the manufacturer, sent out as part of its advertising campaign, stating in relevant part that it will sell the bowling alley owner 40 bowling balls at $10 per ball. A day later, on February 3, the manufacturer received the bowling alley owner’s order. On February 4, the balls were shipped.

On what day did an enforceable contract arise?

A

The contract arose when the balls were shipped. The general rule is that an offer can be accepted by performance or a promise to perform unless the offer clearly limits the method of acceptance. Here, the offer would be the bowling alley owner’s order, because a magazine ad is usually held to be merely solicitation to accept offers rather than an offer. Thus, the manufacturer accepted and the contract was formed when it shipped the balls.

How well did you know this?
1
Not at all
2
3
4
5
Perfectly
35
Q

A young man proposed to his girlfriend, but she was reluctant because of his meager income and lack of job potential. The young man told his father about her reluctance. The father told the girlfriend that if she married his son, he would support them for six months and send his son to a six-month computer technology training school. This was sufficient to dispel her reservations and the two were married very soon after. When they returned from their honeymoon, the father refused to go through with his offer. Although the girlfriend is happy in her marriage, she sued the father for damages.

If the father prevails, what is the likely reason?

Responses

A The father’s promise was not supported by valid consideration.

B The contract is against public policy.

C The contract was oral.

D The girlfriend is happy and therefore has incurred no detriment.

A

If the father prevails, it will be because his promise to the young woman was not in writing. The Statute of Frauds requires that a contract in consideration of marriage must be evidenced by a writing to be enforceable. This includes any promise that induces someone to marry by offering something of value.

How well did you know this?
1
Not at all
2
3
4
5
Perfectly
36
Q

A public high school’s drug policy strictly prohibited the use, possession, or sale of any drug on school grounds, including any prescription or over-the-counter medication, unless supervised by a nurse. During lunch, the school principal observed a student ingesting two white pills. The student admitted to the principal that the pills were aspirins and had been given to her by a senior. School officials approached the senior and demanded to search her backpack. When no aspirins were found in the backpack, the officials required the senior to submit to a private physical search by the female school nurse. Some aspirins were subsequently found in the waistband of the senior’s gym shorts that she was wearing under her school uniform, and she was suspended. The senior’s mother sued school officials, claiming that the physical search violated her daughter’s Fourth Amendment rights against unreasonable searches and seizures. In response, the school officials filed a motion for summary judgment against the mother’s claim.

The facts above are stipulated to by the parties. Should the court grant the motion for summary judgment?

A

The motion should be denied. A school search will be upheld only if it offers a moderate chance of finding evidence of wrongdoing, the measures adopted to carry out the search are reasonably related to the objectives of the search, and the search is “not excessively intrusive in light of the age and sex of the student and the nature of the infraction.”

How well did you know this?
1
Not at all
2
3
4
5
Perfectly
37
Q

A man and a woman were arrested and charged with a series of armed robberies. Each suspect was given Miranda warnings, and different interrogation teams questioned each suspect separately. Upon being questioned, the man told the police, “I’m not going to talk until I see a lawyer.” An officer responded, “You might want to reconsider, because your partner has already confessed, and she’s implicated you in the crimes.” The man then told the police that he wanted to talk to the woman privately. The police escorted the man to the woman’s cell, locked him in with her, and left. Unbeknownst to either of them, the police had bugged the woman’s cell and recorded both the man and the woman making self-incriminating statements during their meeting. The man made no further statements to the police on advice of counsel, whom he called immediately after his conversation with the woman. The man was put on trial first, and the prosecution sought to introduce into evidence tapes of the bugged conversation between the man and the woman. The defense made a motion to suppress the evidence.

Should the court grant the motion to suppress?

Responses

A Yes, because the evidence is the fruit of a wiretap that violated the Fourth Amendment.

B Yes, because the police created a situation likely to induce the defendant to make an incriminating statement.

C No, because there is no expectation of privacy in a jail cell.

D No, because the conversation constituted a waiver of the man’s Miranda rights.

A

The conversation should be suppressed because the police conduct violated the man’s Sixth Amendment right to counsel. The Sixth Amendment provides that in all criminal prosecutions a defendant has a right to the assistance of counsel at all critical stages after formal proceedings have begun. For Sixth Amendment purposes, a criminal prosecution begins when adversary judicial proceedings have commenced, such as the filing of formal charges in this case. Because custodial interrogation is a critical stage of prosecution, the Sixth Amendment is violated by post-charge interrogation unless the defendant has waived his right to counsel. Interrogation includes not only direct questioning, but also any other conduct by the police intended to elicit a response. The police conduct here (telling the man that the woman had implicated him and then bugging the conversation) constitutes prohibited interrogation.

How well did you know this?
1
Not at all
2
3
4
5
Perfectly
38
Q

Which of the following is not an exception to the general rule under the Commerce Clause prohibiting states from discriminating against out-of-state competition?

A
When the regulation is necessary to further an important, noneconomic state interest such as health or safety.

B
When the regulation furthers an important, noneconomic state interest such as health or safety and there are no reasonable alternatives available.

C
When the regulation limits access to privately owned in-state products.

D
When the state acts as a market participant.

A

A regulation limiting access to privately owned in-state products would violate the negative implications of the Commerce Clause; there is no exception for such regulations.

How well did you know this?
1
Not at all
2
3
4
5
Perfectly
39
Q

The Federal Communications Commission (“FCC”) issued a lengthy set of regulations regarding personal radar detectors. The regulations deal with the safety of such detectors and the frequencies on which they may operate, so as not to interfere with FCC-licensed radio and television stations or with radar used by commercial airliners and private aircraft.

May a state constitutionally ban the use of radar detectors on its roads?

Responses

A No, because the regulation of radio transmissions is within the purview of the FCC rather than the states, and state laws that attempt to regulate devices such as radar detectors are preempted.

B No, because such a ban would burden interstate commerce.

C Yes, because such a ban would operate only within the state.

D Yes, because the state has a legitimate interest in regulating the use of radar detectors in order to promote safe driving.

A

The state may ban the radar detectors. States may regulate local aspects of interstate commerce as long as the local regulation does not conflict with, or is not preempted by, federal regulation and the regulation meets the following tests: (i) the regulation does not discriminate against out-of-state competition in order to benefit local economic interests, and (ii) the incidental burden on interstate commerce does not outweigh the local benefits of the regulation. In this case, the federal regulations do not conflict with the state ban and are not so comprehensive as to preempt nonconflicting state regulation.

40
Q

If a state tax on interstate commerce discriminates against a natural person who is a nonresident, which of the following Clauses is least likely to be relevant in determining whether the tax is valid?

Responses

A
The Privileges and Immunities Clause of Article IV

B
The Equal Protection Clause

C
The Privileges or Immunities Clause of the Fourteenth Amendment

D
The Commerce Clause

A

The Privileges or Immunities Clause of the Fourteenth Amendment would not apply to a state tax on interstate commerce that discriminates against a natural person who is a nonresident. It applies when a state denies its own citizens rights of national citizenship.

41
Q

Commercial fishing has long been one of the major industries of a coastal state. To protect the fishing industry and to promote the general welfare of the state’s citizens, the legislature of the state enacted statutes requiring licenses for commercial fishing. An applicant for the license must pay a $300 fee and establish that he has been engaged in commercial fishing in the waters of the state for 10 years. A commercial fisherman residing in a neighboring state frequently takes his fishing boat up the coast. His favorite spot is approximately two miles off the coast of the legislating state.

If the commercial fisherman challenges the constitutionality of the legislating state’s statutes, should the court find the statutes constitutional?

Responses

A Yes, because Congress has not enacted legislation regarding the subject matter of the statutes.

B Yes, because economic and social regulations are presumed valid.
C No, because less restrictive means are available.

D No, because Congress has exclusive power to regulate foreign commerce, which includes commercial ocean fishing.

A

The court should not find the statutes constitutional, because less restrictive means are available. The statutes violate the Privileges and Immunities Clause of Article IV, which prohibits discrimination against nonresidents with respect to essential activities (e.g., pursuing a livelihood) unless (i) the discrimination is closely related to a substantial state purpose, and (ii) less restrictive means are not available. Here, other controls could be placed on fishing without discriminating against out-of-state fishermen.

42
Q

A state law prohibits physicians from practicing medicine within the state without a state license. Among other things, the grant of a state license requires a physician to have been a resident of the state for at least one year. A physician moved to the state from a nearby state and immediately applied for a license to practice medicine. Although otherwise qualified, the physician’s request for a license was denied based on the residency requirement. The physician brought suit, alleging that the residency requirement violated the United States Constitution.

Will the physician likely succeed?

Responses

A Yes, because the requirement violates the Privileges and Immunities Clause of Article IV.

B Yes, because the requirement violates the Privileges or Immunities Clause of the Fourteenth Amendment.

C Yes, because the requirement violates the Due Process Clause of the Fourteenth Amendment.
D No, because the state has a compelling interest in furthering the welfare of its residents.

A

The physician will succeed. The Privileges or Immunities Clause of the Fourteenth Amendment prohibits states from denying their citizens the privileges and immunities of national citizenship. This includes the right to travel, and the Court has held that the right to travel includes the right of newly arrived citizens to enjoy the same privileges and immunities as are enjoyed by other citizens of the state. A state law that distinguishes between new residents solely on the length of their residency will serve no legitimate state interest. Thus, a law limiting medical licenses to persons who have resided in the state for a year runs afoul of the clause.

43
Q

A state located in the southern half of the United States experienced a strong influx of retirees, due in part to its mild winters and in part to the generous health benefits that the state historically provided to its elderly residents who fell below the federal poverty line. The state’s Office of Budget Management determined that the influx of retirees would bankrupt the state’s health care benefit fund within five years. To preserve the fund and ensure the health of its citizens, the state revised its health care statute to make persons ineligible for coverage until they have lived in the state for at least one year.

If a retiree who was denied benefits because she just moved to the state challenges the constitutionality of the statute in federal court, is she likely to prevail?

Responses

A No, because the state has a compelling interest in maintaining the fiscal integrity of its health care fund.

B No, because the states do not have a constitutional duty to provide health care benefits to retirees even if they fall below the federal poverty line.

C Yes, because the requirement improperly burdens the right of interstate travel in violation of the Equal Protection Clause of the Fourteenth Amendment.

D Yes, because the requirement deprives some retirees of certain privileges and immunities in violation of the Privileges and Immunities Clause of Article IV.

A

The court will likely find that the one-year residency requirement is unconstitutional because it burdens the right to travel. An individual has a fundamental right to travel from state to state, and a state law that is designed to deter persons from moving into the state is likely to violate the Equal Protection Clause (as well as the Fourteenth Amendment Privileges or Immunities Clause). When a state uses a durational residency requirement (a waiting period) for dispensing benefits, that requirement normally should be subject to the strict scrutiny test, and usually will be found not to have satisfied the test.

44
Q

For many years, civil service rules have provided that any member of the city’s police department must serve a one-year probationary period before he or she will be considered a permanent employee. However, because the rules were enacted before the city’s police academy was established, a prospective police officer now spends six months in the academy before being hired by the city. A graduate of the police academy was with the city police department for eight months after graduation when she was terminated. There were no city ordinances or state laws that required that she be given either a reason for the termination or a hearing, and she was given neither. The graduate brought suit against the city in state court because of the termination of her employment, alleging a violation of her due process rights.

Which of the following would most likely give her a constitutional basis to require the city to give her a statement of reasons for the termination and an opportunity for a hearing?

Responses

A No police officer had ever been terminated during probation except where there was actual cause.

B The six months she spent in the academy must be considered as part of her probationary period.

C The budget of the police department was recently increased to allow for the hiring of additional officers.

D She was the only female police officer on probation and the only officer not given permanent employment.

A

The fact that no police officer has been terminated during probation except for cause may be enough for the graduate to show that she has a right to a hearing. Continued public employment may be a protected property interest if there is a clear practice or mutual understanding that an employee can be terminated only for “cause.” If the graduate can establish this, she will be able to force the city to give her a reason for her termination and a hearing.

45
Q

The police department of a small city has jurisdiction within the city limits and over a defined portion of the surrounding rural communities within the county. A farmer lives in one of the rural communities receiving police protection from the city. The farmer does not pay any tax to the city directly, but a portion of the farmer’s county property tax is turned over by the county to the city in order to support the city’s police department.

The farmer’s property was vandalized several times over the past several months, and the farmer became unhappy with the police protection that the city was providing. After his complaints to the police department and city hall did not improve the situation, the farmer wanted to vote against the mayor in the next election, but a city ordinance provides that only residents of the city may vote in city elections.

If the farmer brings a suit to compel the city to allow him to vote in the city’s mayoral election, is he likely to prevail?

Responses

A No, because the resident voting limitation appears to be rationally related to a legitimate government interest.

B No, but only if the city can prove that the resident voting limitation, which affects a fundamental right, is necessary to a compelling interest.

C Yes, because the resident voting limitation violates the Privileges or Immunities Clause of the Fourteenth Amendment.

D Yes, because the resident voting limitation constitutes an instance of taxation without representation.

A
46
Q

Which of the following is not itself sufficient to trigger strict or intermediate scrutiny on a claim that government action discriminates on the basis of a suspect or quasi-suspect classification?

Responses

A
Facial discrimination

B
Discriminatory motive

C
Discriminatory effect

D
Discriminatory application

A

The discriminatory effect of governmental action is not enough, standing alone, to trigger strict or intermediate scrutiny. Only intentional discrimination violates the Equal Protection Clause, and the mere fact that government action appears to have a discriminatory effect does not show a discriminatory intent.

47
Q

man was a permanent resident alien of the United States who was awaiting an opportunity to become a citizen. He filed an application to become an instructor in the local public high school but was denied the position solely on the ground that he was not a citizen. The man now brings suit, alleging that his status as a resident alien was not a proper ground for denying him a position as an instructor.

May the state deny a permanent resident alien employment as an instructor in the public high school?

Responses

A Yes, because employment by the state is a privilege, not a right.

B Yes, because citizenship bears some rational relationship to the interest that is being protected.

C No, to do so would be a denial of equal protection.
D No, because the evidence presented was uncontroverted that he was awaiting the opportunity to become a citizen.

A

The state’s action would be reviewed under the rational basis standard. Although state classifications based on alienage are generally suspect, a state may reserve a government position for citizens if it is related to self-governance, involves policymaking, or requires exercise of important discretionary power over citizens. In these cases, only a rationality test is used. A public school teacher at the primary and secondary school level performs an important governmental function (e.g., he influences students’ attitudes about government, the political process, citizenship, etc.), and therefore the exclusion of aliens is rationally related to the state’s interest in furthering educational goals.

48
Q

Among the categories of speech that are not protected by the First Amendment is “fighting words.”

Which of the following statements is not true when it comes to regulation of fighting words?

Responses

A
Fighting words-words or epithets that, when addressed to an ordinary citizen, are inherently likely to incite immediate physical retaliation-may be punished.

B
Fighting words statutes are often struck down for overbreadth.

C
True threats-statements meant to communicate an intent to place an individual or group in fear of bodily harm-may be punished.

D
Hate crime statutes may limit fighting words sanctions to cases in which the words seek to insult or provoke on the basis of race, religion, or sexual orientation.

A

It is not true that hate crime statutes may limit fighting words sanctions to cases in which the words seek to insult or provoke on the basis of race, religion, or sexual orientation. Such a limitation means the sanction is based on viewpoint, and the Court will not tolerate such sanctions.

49
Q

Question
A city zoning board recently denied a request from the local library for a variance needed to expand the library building. An angry library patron went to the library, stood next to the front door, and handed each person entering a leaflet asking the person to contact each city zoning board member named in the leaflet and threaten to vote the member out of office unless the member changed his or her vote regarding the library’s request for a zoning variance. The head librarian noticed the patron handing out the leaflets and asked her to stop, correctly explaining that the distribution was in violation of a city ordinance. The patron refused to comply, and the head librarian summoned the police. When an officer arrived, the librarian again asked the patron to stop distributing leaflets, but the patron again refused. The officer then arrested the patron for violating a city ordinance. At trial, the patron defended against the charges by claiming a violation of her First Amendment rights.

Which of the following variations of fact would be most helpful to the patron’s First Amendment claim?

Responses

A The librarian did not repeat the “cease and desist” request in the police officer’s presence.

B The library is completely surrounded by public sidewalks.

C The librarian has permitted some people to distribute leaflets at the front door of the library at all hours.

D The leaflets also requested campaign contributions to be used to oppose board members who failed to change their vote.

A

The most helpful additional fact for the patron is that the head librarian allowed other people to distribute leaflets at all hours. Although the government may adopt reasonable time, place, and manner restrictions in public forums and designated public forums, such restrictions must be content-neutral, or else they will be subject to strict scrutiny. The head librarian’s allowing some people to distribute leaflets at all hours shows that the restriction here is probably being used as a content regulation, meaning that the restriction would be subject to strict scrutiny and likely invalidated. Additionally, if the head librarian is allowing others to distribute leaflets at all hours, the discriminatory application of the ordinance might also violate the Equal Protection Clause.

50
Q

A state statute makes it a felony for anyone in the corridors or on the grounds of any building in which a court may be in session to make a speech or carry a sign intended to improperly influence judicial proceedings. When the head of a street gang was on trial for murder, a gang member was arrested for carrying a sign on the steps of the courthouse warning that if the gang leader was not freed, “the judge will die.”

May the gang member be convicted of violating the state statute?

Responses

A No, because the statute could apply to others whose speech is constitutionally protected.

B No, unless he personally intended to harm the judge.

C Yes, if there was a clear and present danger that the judge would be influenced by the sign.
D Yes, because the statute does not violate the freedom of expression guaranteed by the First Amendment.

A

The gang member can be convicted because the statute does not violate the First Amendment. Certain public property (e.g., public streets or parks) is so historically associated with the exercise of First Amendment rights that speech thereon can be regulated only by content-neutral proscriptions. Other places controlled by the government, however, are not so historically linked to speech activities, and in such locations free speech might interfere with the intended use of such locations. Thus, the government can regulate access to these limited public forums and nonpublic forums based on the subject matter of the speech, as long as the regulation is reasonably related to the purpose served by the property and is not designed merely to suppress a particular point of view. A courthouse and its grounds are not a public forum. (The surrounding sidewalks are, but that is not in issue here.) The statute, although based on the subject matter of speech, is viewpoint neutral and reasonably related to the courthouse purpose of promoting a stable, orderly atmosphere in which judicial proceedings can take place, free of improper outside influence or coercion. Thus, the statute is valid and the gang member can be convicted for his actions.

51
Q

At the defendant’s trial for murder, facts were introduced that the defendant acted in the heat of passion. After a lengthy trial, the defendant was convicted of manslaughter. On appeal, the conviction was reversed on procedural grounds. The state immediately moved to retry the defendant, again bringing murder charges against her. The defendant moved to strike the murder charge, and the court refused to grant the motion. After the second trial, the defendant was again convicted of the lesser charge of manslaughter. The defendant appeals the second conviction, claiming that it violated her constitutional rights.

May the second conviction stand?

Responses

A No, because the state could not refile charges after the acquittal.

B No, because the state could not retry the defendant for murder under the circumstances.

C Yes, because the state could retry the defendant because the manslaughter conviction was overturned.

D Yes, because the defendant was reconvicted of the lesser charge again, so any error was harmless.

A

The second conviction will not stand. The Double Jeopardy Clause prohibits retrying a defendant whose conviction has been reversed on appeal for any offense more serious than that for which she was convicted at the first trial. This right is violated by retrial for a more serious offense, even if at the second trial the defendant is only convicted of an offense no more serious than that for which she was convicted at the first trial.

52
Q

A woman was the subject of a murder investigation. The investigation continued for more than two years, with the woman frequently being called in for questioning. Finally, the woman was indicted for the murder. The woman’s lawyer filed a motion to dismiss all charges against her, arguing that the excessively long investigatory period violated the woman’s constitutional right to a speedy trial.

Despite the pending motion, the woman decided that she wanted to “get it over with,” and she told the judge that she wished to plead guilty. The judge then explained the charges to the woman and asked her if she understood them. She replied, “Yes.” The judge then asked the woman if she understood that she was not required to plead guilty. She responded in the affirmative. Finally, the judge described the maximum sentence and asked the woman if she understood that she could receive the maximum sentence, which was life imprisonment. She again responded, “Yes,” and maintained that she still wished to plead guilty. The judge accepted the woman’s plea and sentenced her to 30 years’ imprisonment in the state penitentiary. Six months later, the woman filed a motion to set aside the guilty plea.

Which of the following provides the best argument that the woman has a constitutional basis for relief?

Responses

A The judge did not rule on the pending motion to dismiss before accepting her guilty plea.
B The judge did not attempt to determine if the woman had actually committed the murder.

C The judge did not determine whether the files in the prosecutor’s office contained any undisclosed exculpatory evidence.

D The judge did not determine whether the woman understood that she had a right to a trial by jury.

A

The judge’s failure to determine whether the woman understood her right to trial by jury indicates that her guilty plea does not satisfy the constitutional requirement that it be “voluntary and intelligent.” A guilty plea is a waiver of the Sixth Amendment right to a jury trial. To be a valid waiver, the judge must determine on the record that the guilty plea represents a voluntary and intelligent choice among the alternative courses of action open to the defendant. To ensure that this is the case, the judge should make sure that the defendant is informed of the nature of the charge to which the plea is offered, of the maximum possible penalty, that she has a right not to plead guilty, and that by pleading guilty she waives her right to a trial. If the judge did not determine whether the woman understood that she had a right to a trial by jury, her plea will not be a sufficiently intelligent choice to satisfy the constitutional standard, and therefore will not be immune from a post-sentence attack on it.

53
Q

A man shopping for a leather jacket at a clothing store could not decide between two jackets, so the proprietor, who knew the man and his family well, let him take one of the jackets on approval. No mention was made by the proprietor of the method of payment he expected. The man wore the jacket on a visit to his grandfather, who liked it so much that when the man told him what the jacket cost and that he had taken it on approval, the grandfather said he would buy it for him if he promised to give some of his old clothes to a favorite charity for the poor at Christmastime. The man wholeheartedly agreed to donate the clothes to the charity at Christmas. Very pleased, the grandfather called the shop and told the proprietor to send the bill for the jacket to him, which he did. Before the bill was paid and before the Christmas season arrived, the grandfather fell ill and died. The grandfather’s executor has refused to pay the bill, and the man has not yet given any old clothing to the charity.

Will the proprietor be able to recover the price of the jacket from the estate?

Responses

A Yes, because the proprietor was the intended beneficiary of the promise between the man and his grandfather.

B Yes, because the man has no duty to give the clothing to the charity.

C No, because the grandfather’s implied promise to pay the proprietor arising from the phone call is unenforceable.

D No, because a condition has not yet occurred.

A

The proprietor can recover the cost of the jacket from the grandfather’s estate because the proprietor is an intended third-party beneficiary and his right to enforce the contract has vested. The rights of an intended third-party beneficiary vest when the beneficiary (i) manifests assent to the promise in a manner invited or requested by the parties; (ii) brings suit to enforce the promise; or (iii) materially changes his position in justifiable reliance on the promise. Here, the proprietor qualifies as an intended beneficiary of the agreement between the man and his grandfather because the proprietor was expressly designated in the contract, he was to receive performance directly from the grandfather, and he stood in an existing contractual relationship with the man that required the man to either pay for the jacket or return it, making it likely that the young man’s purpose in making the arrangement with his grandfather was to satisfy the obligation to the proprietor. The proprietor can enforce the contract because his rights vested when he sent the bill to the grandfather at the grandfather’s request.

54
Q

A landowner entered into a written agreement with a real estate broker whereby the broker would receive a commission of 10% of the sale price if he procured a “ready, willing, and able buyer” for the landowner’s property and if the sale actually proceeded through closing. The broker found a buyer who agreed in writing to buy the property from the landowner for $100,000, the landowner’s asking price. The buyer put up $6,000 as a down payment. The agreement between the landowner and the buyer contained a liquidated damages clause providing that, if the buyer defaulted by failing to tender the balance due of $94,000 at the closing date, damages would be 10% of the purchase price. The landowner included that clause because she was counting on using the proceeds of the sale for a business venture that would likely net her at least $10,000.

The buyer became seriously ill and defaulted. When he recovered, he demanded that the landowner return his $6,000, and the landowner refused. The broker also demanded the $6,000 from the landowner and was refused. The broker and the buyer filed separate suits against the landowner, with the buyer pleading impossibility of performance. The two cases are consolidated into a single case.

How should the court rule as to the disposition of the $6,000?

A

The landowner may keep the $6,000 as liquidated damages. A liquidated damages clause is enforceable if: (i) damages are difficult to ascertain at the time of the making of the contract, and (ii) the damages are a reasonable forecast of compensatory damages. Here, the landowner was unsure what her damages would be if she did not receive the sales proceeds from the property, but $10,000 seemed a reasonable amount.

55
Q

A state’s civil procedure rules allow for “nail and mail” service (posting the summons at the defendant’s dwelling and thereafter mailing him a copy by certified mail) when “regular” service (service by physical delivery or by leaving a copy of the complaint with someone of suitable age and discretion at the defendant’s usual place of abode) cannot be accomplished with due diligence. A plaintiff brought suit against a defendant in the federal district court for that state. After the plaintiff’s special process server made many attempts at serving the defendant, the plaintiff’s attorney directed the process server to nail the complaint and summons to the defendant’s front door, and the attorney mailed (by certified mail) a copy to the defendant in accordance with the state rule.

Has the defendant been properly served?

Responses

A Yes, if “nail and mail” service is reasonably calculated to give the defendant notice of the action.

B Yes, because under the Erie doctrine the federal court must apply the state’s service of process rules.

C No, if the defendant only rented the dwelling at which service was posted.

D No, because “nail and mail” service is not authorized by the Federal Rules of Civil Procedure.

A

Federal Rule 4 provides that summons and complaint may be served on an individual other than an infant or incompetent pursuant to the law of the state in which the district court is located. Notwithstanding, the state provision must be constitutional; i.e., it must be reasonably calculated to give the defendant notice of the action.

56
Q

The defendant and an accomplice were on trial together for burglary. Both had given confessions implicating themselves and their accomplice. At trial, the defendant maintained that his confession had been obtained through improper coercion by the police. For the purpose of countering the claim of coercion, the prosecution seeks to place the accomplice’s confession into evidence. After objection by the defendant’s counsel, the judge agrees to issue a limiting instruction to the jury that the confession is to be considered only with regard to the question of whether the defendant’s confession was coerced.

May the accomplice’s confession be admitted under that condition?

A

The confession is admissible with the judge’s limiting instruction. Where two persons are tried together and one has given a confession implicating the other, the general rule is that the Sixth Amendment right to confront adverse witnesses prohibits the use of such a statement. This problem arises because of the inability of the nonconfessing defendant to compel the confessing co-defendant to take the stand for cross-examination at their joint trial. As exceptions to the general rule, the statement may be admitted if: (i) all portions of the statement referring to the other defendant can be eliminated (so that there is no indication of that defendant’s involvement); (ii) the confessing defendant takes the stand and subjects himself to cross-examination with respect to the truth or falsity of what the statement asserts; or (iii) the confession of the nontestifying co-defendant is being used to rebut the defendant’s claim that his confession was obtained coercively, in which case the jury must be instructed as to the purpose of the admission.

57
Q

A defendant charged with driving while intoxicated pleaded not guilty and insisted on a trial. Right before the trial began, he fired his attorney and decided to defend himself. At one point during opening arguments, the defendant began to act like a cat, meowing and chasing an imaginary squirrel out of the courtroom.

If no one else raises the issue of the defendant’s competency to stand trial, what is the responsibility of the trial judge here?

Responses

A The trial judge has no responsibility, because the defendant decided to defend himself.

B The trial judge has no responsibility, because she cannot decide whether the defendant is competent to stand trial.

C The trial judge must raise the issue of competency, because the defendant is representing himself.

D The trial judge must raise the issue of competency, because the Constitution obligates her to do so.

A

The judge must raise the issue of competency. If it appears to the judge that the defendant might be incompetent, the judge has a constitutional obligation to conduct further inquiry and determine whether in fact the defendant is incompetent. If the defendant is tried and convicted but it later appears that he was incompetent to stand trial, the judge’s failure to raise the issue or to request a determination of competency does not constitute a waiver of the competency issue. [Pate v. Robinson (1966)] Therefore, if the trial judge observes the defendant acting in such a way that may indicate he is incompetent to stand trial (e.g., meowing, chasing imaginary squirrels), she should conduct further inquiry to determine the competency of the defendant.

58
Q

The police received a tip from a reliable informant that a former student at the local university was selling narcotics. A brief investigation revealed that the former student, a college dropout, still hung around the university campus, had no visible means of support, and yet drove a large luxury car and wore flashy clothing and jewelry. The police picked up the former student the next time he showed up on campus, took him to the station, and questioned him all night long without a break and without letting him communicate with anyone else. When the former student tired from the interrogation, he admitted that he sold cocaine to his friend, who is a current student at the university. Based on this information, the police went to the current student’s dormitory room. When they arrived, they found the door open but no one was in the room. The police entered, searched the room, and discovered a vial of white powder. Later laboratory tests established the powder to be cocaine. The former student was then charged with the sale of narcotics. At his trial, the prosecution attempted to admit the cocaine discovered in the dormitory room into evidence.

What is the former student’s best argument for preventing the cocaine from being admitted into evidence?

Responses

A The search of the dormitory room was conducted without a warrant and without consent.

B The police arrested the former student without a warrant.

C The former student’s confession was not voluntary under the circumstances.

D The police failed to give the former student Miranda warnings.

A

The former student’s best argument for preventing the cocaine from being admitted into evidence is that his confession was not voluntary. Choices (C) and (D) both focus on improper conduct during the former student’s interrogation, but (C) is better because the former student will have a better chance of invoking the exclusionary rule if the confession is involuntary. For confessions to be admissible, the Due Process Clause of the Fourteenth Amendment requires that they be voluntary. While voluntariness is a fact question that is assessed by looking at the totality of the circumstances, the duration and manner of the police interrogation here indicate that the confession probably was the result of actual coercion. If the confession is found to be involuntary, the former student can invoke the exclusionary rule to exclude the cocaine as “fruit of the poisonous tree.”

59
Q

A small processor of specialized steel agreed in writing with a small manufacturer of children’s toys that it would supply, and the manufacturer would buy, all of the manufacturer’s specialized steel requirements over a period of years at a set price per ton of steel. Their contract did not include a nonassignment clause. Recently, the toy manufacturer decided to abandon its line of steel toys, so it made an assignment of its rights and delegation of its duties under the contract to a toymaker many times larger. The large toymaker notified the steel processor of the assignment and relayed to the processor its good faith belief that its requirements will approximate those of the assignor.

Must the steel processor supply the requirements of the large toymaker?

Responses

A Yes, because there was no nonassignment clause in the contract.

B Yes, because the large toymaker acted in good faith to assure the steel processor that its requirements will approximate those of the small manufacturer into whose shoes it stepped.

C No, because requirements contracts are not assignable under the UCC

D No, because the steel processor did not give prior approval of the assignment.

A

Generally, the right to receive goods under a requirements contract is not assignable because the obligor’s duties could change significantly. In fact, here, a significant change would seem possible because the large toymaker is a larger company than the small manufacturer and its needs could be greater. However, the UCC allows the assignment of requirements contracts if the assignee acts in good faith not to alter the terms of the contract. [UCC §2-306] (The UCC applies here because goods are involved.)

60
Q

A defendant is charged with beating a victim to death with a set of brass knuckles during the course of a fight in a tavern. The victim was found to have a pistol on his person at the time of the fight. During the course of the trial, the defendant took the stand in his own defense and testified that the victim threatened him with a gun and the defendant had hit the victim with the brass knuckles in self-defense. To rebut the defendant’s claim, the prosecution wishes to place the bartender on the stand, who will testify that two years prior to the attack on the victim, she had seen the defendant approach a customer in her tavern from behind, put on a pair of brass knuckles, and strike the customer a severe blow on the side of the face with a brass-knuckled fist. The prosecutor has complied with all notice requirements regarding the bartender’s proposed testimony. As soon as the bartender is sworn in, the defense attorney raises an objection.

How should the court rule on the admissibility of the bartender’s testimony?

Responses

A Admissible, as substantive evidence that the defendant did not act in self-defense in beating the victim.

B Admissible, to attack the defendant’s credibility.

C Inadmissible, because prior bad acts cannot be admitted to prove the defendant’s propensity to commit the specific crime with which he is charged.

D Inadmissible, because the defendant has not put his character in issue in this case.

A

The bartender’s testimony is inadmissible because the defendant’s prior fight in the tavern cannot be admitted to prove his propensity to beat someone to death. The basic rule is that when a person is charged with one crime, extrinsic evidence of his other crimes or misconduct is inadmissible if such evidence is offered solely to establish a criminal disposition. [Fed. R. Evid. 404(b)] The danger is that the jury may convict because of past conduct rather than because of guilt of the offense charged. While evidence of other crimes is admissible if it is independently relevant to some other issue (e.g., motive, intent, or identity), the defendant’s prior fight appears to have no relevance other than as evidence of his violent disposition. It is therefore inadmissible.

61
Q

The plaintiff, an electrical contractor, sued the defendant homeowner for refusal to pay for extensive wiring repairs performed on his home by the plaintiff’s employee. The plaintiff called the employee to the stand. The employee, under oath, testified that he did not perform any work at the defendant’s home. The employee also denied writing a letter to a friend telling the friend that the employee was going to do electrical work on the home. Without releasing the employee as a witness, the plaintiff offers into evidence the letter written by the employee to his friend.

If the employee’s letter to his friend is properly authenticated, should the court admit the letter?

Responses

A Yes, for impeachment purposes only.

B Yes, as both substantive and impeachment evidence.

C No, because a party may not impeach his own witness.

D No, because it is inadmissible hearsay.

A

The letter is admissible as substantive evidence as well as for impeachment purposes. For the purpose of impeaching the credibility of a witness, a party may show that the witness has, on another occasion, made statements that are inconsistent with some material part of his present testimony. This may be done by first questioning the witness as to the prior inconsistent statement that he has made. If the witness denies having made the statement or fails to remember it, the making of the statement may be proved by extrinsic evidence. A proper foundation must be laid by giving the witness an opportunity to explain or deny the statement, and it must be relevant to some issue in the case. Under Rule 803(3), a statement of a declarant’s then-existing state of mind is admissible as a basis for a circumstantial inference that the declarant acted in accordance with his state of mind. [See also Mutual Life Insurance Co. v. Hillmon (1892)] The employee’s statement that he was going to do electrical work on the home is admissible as circumstantial evidence tending to show that he followed through with his plans and did the electrical work, which is what the statement is being offered to establish. In this case, therefore, the letter should be admissible as both substantive and impeachment evidence,

62
Q

A defendant is on trial for robbing a liquor store. The store clerk testified that the defendant came into the store at about 11 p.m., pointed a black gun with a silver grip at him, and demanded that he give him all the money in the cash register. The clerk testified that the store was well lit and that the defendant was not wearing a mask. The defendant’s attorney called the clerk’s employer to testify that when the clerk gave her a report of the robbery, he told her that the defendant pointed a silver gun with a black grip at him.

How should the trial judge rule on the admissibility of this testimony?

Responses

A Admissible, because it tends to show that the clerk is an unreliable witness.

B Admissible, because it has bearing on the clerk’s truthfulness and veracity.

C Inadmissible, because it is extrinsic evidence of a prior inconsistent statement on a collateral matter.

D Inadmissible, because it is hearsay not subject to any exceptions.

A

The testimony should be found inadmissible. Extrinsic evidence of a prior inconsistent statement may not be used to impeach a witness upon a collateral matter. The clerk testified that he could recognize the defendant’s face, and so the color of the gun is not material to any issue in the case under the facts given.

63
Q

An elderly grandfather who wanted to ensure that his property would remain in the family after his death included the following clause in his will: “I give my house in the city to my son, but if he ever tries to sell it while he is alive, I want it taken away from him and given to my grandson.” The grandfather’s will was properly executed.

When the grandfather later died, what interests did the son and grandson take in the property?

Responses

A The son took a fee simple.The son took a fee simple.

B The grandson took a fee simple.The grandson took a fee simple.

C The son took a fee simple subject to an executory interest, and the grandson took an executory interest.

D The son took a fee simple determinable, and the grandson took a contingent remainder.

A

The son received a fee simple in the property. The grandfather attempted to give his son a fee simple, but placed a restraint on alienation. Direct restraints on alienation of a fee simple are void. The grant is simply read as if it had been “O to A in fee simple.” The grandson gets nothing. Thus, (A) is correct.

64
Q

A woman owned 400 acres of land, half of which was densely wooded and the other half of which was almost entirely occupied by a large gravel pit which supplied gravel for her small landscaping business. A small house located on the edge of the pit was in very poor condition and had been vacant for many years. The woman transferred the 400 acres to her son for life, with the remainder going to a local charity on the son’s death. Now the son wants to increase gravel production and expand the pit by tearing down the house. He also wants to cut the trees on the wooded half and sell them for profit. The local charity, holder of the remainder, sues to enjoin the son from doing any of these things.

How will the court likely rule?

Responses

A The charity can stop both the gravel mining and the tree cutting and can block the destruction of the house.

B The charity can stop neither the gravel mining nor the tree cutting but can block the destruction of the house.

C The charity can stop the tree cutting but not the gravel mining or the destruction of the house.

D The charity can stop the gravel mining and the tree cutting but not the destruction of the house because it is dangerous and unfit for use.

A

The charity can stop only the tree cutting. The charity would be suing the life tenant on a theory of waste. Both the gravel mining and the tree cutting could be voluntary waste. Generally, a life tenant can only maintain the property and not sell off any of the natural resources, such as trees and gravel. But there is an exception for existing exploitation of these resources. Because the gravel mine was operating prior to the son’s taking of the life estate, that preexisting use is protected and the son is not liable for continuing the mining of gravel. But the trees do not fall under this exception. Because the property had not been in use as a tree farm, the general rule applies and the cutting of the trees would be waste. The destruction of the house might be considered ameliorative waste-the destruction of improvements on the life estate that increase the value of the property (i.e., they make the gravel mining more profitable). Generally, a life tenant cannot tear down improvements simply because the life tenant wants to make a more profitable use of the land. But an exception exists when changed conditions have made the destruction of the improvement reasonably necessary. Here, the house was on the edge of the pit, vacant because of its very poor condition. Changed conditions (the encroachment of the pit) have made the house reasonably unsuitable, so the life tenant can tear it down.

65
Q

A husband and wife held land as joint tenants with right of survivorship. On the marriage of their daughter, the husband and wife conveyed a 10% interest to her. Two years later, the husband and wife conveyed another 10% interest to the daughter’s spouse.

Which of the following best describes how the land is now owned?

Responses

A The husband and wife hold an 80% interest as joint tenants, and the daughter and her spouse each hold a 10% interest as tenants in common.

B The husband and wife hold an 80% interest as joint tenants, and the daughter and her spouse hold a 20% interest as joint tenants.

C The husband has a 40% interest, the wife has a 40% interest, the daughter has a 10% interest, and her spouse has a 10% interest, all as tenants in common.

D The husband and wife each hold a 40% interest as tenants in common, and the daughter and her spouse hold a 20% interest as joint tenants.

A

The husband and wife hold an 80% interest as joint tenants, and the daughter and her spouse each hold a 10% interest as tenants in common. When the husband and wife conveyed 10% to their daughter, each retained 45% as joint tenants with one another, and the daughter’s 10% is, by severance, a tenancy in common. Because both the husband and wife joined in the conveyance to their daughter, their interests remain undisturbed as joint tenants. Similarly, when the husband and wife then conveyed 10% to the daughter’s spouse, they each retained their 40% as joint tenants with one another, and the spouse’s 10% is, by severance, a tenancy in common.

66
Q

Question
The elderly owner of a small furnished vacation cottage who had not used it for several years decided to rent it out to vacationers. His first tenants, a newlywed couple, agreed to pay $400 for a month stay, although the owner apologized that he had not been able to clean the cottage before renting it because he had not been there for several years. When they arrived at the cottage, the wife started carrying the luggage up the wooden steps of the house and one of the steps collapsed, seriously injuring her. On inspection it was revealed that the steps had rotted. Although no one had been aware of the defect, a reasonable inspection would have disclosed it.

Is the owner liable to the wife for her injuries?

Responses

A No, because the owner neither knew nor had reason to know that the steps were rotted.

B No, because the owner told the tenants that he had not been to the house for several years and so was not aware of its current condition, and the wife could have discovered the defect if she had made a reasonable inspection.

C Yes, because a landlord warrants that premises are free of latent defects.

D Yes, because the lease is for one month and the house is furnished.

A

The owner is liable to the wife. Landlords are liable for latent defects even if they neither knew nor should have known of the defect if the lease is for a short term and the property is furnished. This is an exception to the general rule that a landlord is not liable for latent defects unless the landlord either knew or had reason to know of the defects.

67
Q

A tenant entered into a two-year apartment lease with a landlord on July 1. Rent was specified in the lease to be $850 per month, payable on the first of each month.

On June 15, near the end of the two-year term, the landlord asked the tenant if he wanted to renew the lease for an additional term. The tenant said he would like to think about it, and the landlord agreed, but added that the rent for the new lease would be $975 per month. The landlord heard nothing from the tenant. On July 10, the landlord found that the tenant was still in the apartment and told him that he was imposing a new tenancy on the tenant for the period allowed by the law under these circumstances.

What tenancy can be imposed, and at what rent?

Responses

A The landlord can impose a new periodic tenancy of month-to-month and the rent will be $850 per month.

B The landlord can impose a new periodic tenancy of month-to-month and the rent will be $975 per month.

C The landlord can impose a new periodic tenancy of year-to-year and the rent will be $850 per month.

D The landlord can impose a new periodic tenancy of year-to-year and the rent will be $975 per month.

A

The tenancy will be month-to-month at $975 per month. When a tenant wrongfully holds over after the expiration of a lease, the landlord has two choices: either treat the tenant as a trespasser and sue for damages and possession or impose a new periodic tenancy on the hold-over tenant. If the landlord chooses, as this landlord has done here, to impose the new periodic tenancy, most courts in residential situations would impose a month-to-month tenancy. While the rent (as well as other terms) of the new tenancy will generally be the same as the old tenancy, there is an exception when the landlord has told the tenant of a future higher rent and that notification came before the expiration of the old lease. In that event, the landlord can impose the higher rent in the new periodic tenancy. Exam Tip: The facts in this type of question may vary-be sure that the landlord told the tenant of the higher rent before the expiration of the old lease; otherwise, the old, lower rent applies. Because the landlord told the tenant that the tenant could stay on at $975 and this occurred on June 15, prior to the expiration of the two-year lease, then if the tenant held over and the landlord elected to hold the tenant to a new periodic tenancy, it would be at the higher rent.

68
Q

The owner of a house purchased a new home but decided to keep his old residence for a few months until the real estate market improved. He rented it to a tenant with the understanding that the tenant might have to move out in a few months if the house was sold. The tenant paid the owner the agreed rent of $100 per week every Friday.

During the next few months, the owner’s business suffered serious setbacks so he decided to sell his new home and move back into his old one. He informed the tenant that he would have to vacate the old home, but the tenant refused to vacate and tendered the $100 rental payment the following Friday, which the owner refused to accept.

The owner immediately filed suit to eject the tenant. The jurisdiction requires that a statutory written notice be served on any tenant whose term is for less than month-to-month or is not for a fixed term at least three days prior to commencement of eviction proceedings. No written notice of any kind was given to the tenant.

How should the owner characterize the tenant to gain immediate possession of his home?

Responses

A As a tenant from month to month.

B As a tenant from week to week.

C As a licensee.

D As a trespasser ab initio.

A

The owner’s best argument is that the tenant is a licensee. A license is a privilege to enter onto another’s property. It may be revoked at any time merely by a manifestation of the licensor’s intent to end it.

69
Q

A woman owned two adjacent parcels. The east parcel fronts on a poor unpaved public road, while the west parcel fronts on a major highway. Fifteen years ago, the woman conveyed the east parcel to her son “together with a right-of-way 25 feet wide over the north side of the west parcel to the highway.” At that time, the east parcel was improved with a 10-unit motel.

Ten years ago, the woman died. Her will devised the west parcel “to my son for life, remainder to my daughter.”

Five years ago, the son executed a deed purporting to convey the east and west parcels to his friend in fee simple. The friend then enlarged the motel to 12 units.

Six months ago, the son died and the daughter took possession of the west parcel. She brought an appropriate action to enjoin the friend from using the right-of-way.

Who should prevail?

Responses

A The daughter, because merger extinguished the easement.

B The daughter, because the friend has overburdened the easement.

C The friend, because he has an easement by necessity.

D The friend, because he has the easement granted by the woman to her son.

A

The friend should prevail. The deed from the woman to her son granted the son an express easement. That easement was never terminated by merger and the friend, the owner of the motel property on the east parcel, still has a valid easement over the daughter’s property, the west parcel. (A) is incorrect because, for there to be a merger which will extinguish an easement, the duration of the servient estate must be equal to or longer than the duration of the dominant estate (and therefore the easement). In this case, the son owned the east parcel in fee simple but owned only a life estate in the servient estate, the west parcel. Therefore, because the duration of the servient estate was shorter, there was no merger of the two estates and the easement was not extinguished by the son’s partial ownership of the servient estate.

70
Q

Question
A woman who owned a parcel of land had direct access to the main road by an unpaved driveway. However, she decided that it would be more convenient to use a paved driveway on an adjoining parcel, so she began doing so. The man who owned the adjoining parcel also used the driveway, but he did not discover that the woman was using the driveway until two years later. At that time, he wrote a letter to the woman protesting the use of the driveway, but the woman continued to use the road. After she had used the road for a total of 20 years, she filed an action for declaratory judgment, claiming a prescriptive right to use the driveway on the west parcel. The time limit necessary to obtain an easement by prescription in this jurisdiction is 20 years.

Which of the following is the most accurate statement with regard to the woman’s use of the driveway now?

Responses

A The woman could not acquire a prescriptive easement in the west parcel because her original use was wholly trespassory and without any claim of right.

B The prescriptive period began to run when the woman first began using the driveway because the woman was in open and notorious possession, despite the fact that the man did not know of it.

C The prescriptive period began to run when the woman first began using the driveway despite the fact that, at that time, she was sharing her use with the man.

D The prescriptive period did not begin to run until two years after the woman began using the driveway, at the time her use of the driveway was contested.

A

C is the correct answer. The woman’s first use of the driveway started the prescriptive period running. There is no requirement of exclusive use for the woman to obtain an easement by prescription. Exclusivity is only required in cases of adverse possession. Adverse possession is not at issue here because the woman has not “possessed” any part of the west parcel, and does not require title to continue her use of the property, which is all she seeks. No merger because the son had a life estate and a fee simple. Both parcels must be owned in the same manner.

71
Q

Several decades ago, a square tract of 640 acres was subdivided into lots, streets were constructed, and utilities were installed. As the developer sold off each of the lots, each lot’s deed contained a restriction limiting the lots to residential use only. The deeds were all recorded. Over the years, houses were constructed on all of the lots. The property to the south, east, and west of the subdivision was initially forest, but gradually the city expanded to surround the development. Unfortunately, the city expansion was mostly industrial and some commercial property, but no residential property. The subdivision is now bounded on all sides by many industries that operate 24 hours a day. The combination of noise, dirt, fumes, and other pollution has made many of the houses in the subdivision unfit for residential use, yet each deed still stipulates “residential use only.”

Can the restriction be voided under the doctrine of changed neighborhood conditions?

Responses

A Yes, the restriction can be voided on all lots in the subdivision because changed conditions have made many of the houses unusable as residences.

B Yes, the restriction can be voided under these circumstances, but only as to the lots that have actually been rendered unfit for residential purposes.

C No, the restriction cannot be voided because in this circumstance the proper remedy is the tort action of nuisance.

D No, the restriction cannot be voided unless the entire subdivision is so seriously affected by the pollution that enforcement of the restriction would be inequitable.

A

The restriction cannot be voided under the doctrine of changed neighborhood conditions. Restrictive covenants on all lots in a subdivision such as this can be voided if changed conditions have made the property unusable for the specified use, and this means that the entire subdivision must have changed so significantly that enforcement of the restriction would be inequitable. If some houses in the center of the subdivision are not affected by the pollution, then none of the restrictions can be voided; if all lots are affected, then all restrictions are voided.

72
Q

A landowner contracted in writing to sell a lot to a brother and a sister, as joint tenants, for $60,000. The brother and the sister put up $6,000 as earnest money. Before the closing date, the landowner died. Shortly thereafter, and also before the closing date, a garage on the lot burned down. The garage had a fair market value of $6,500 and was a complete loss. After the fire, the brother demanded that the executor of the landowner’s estate return the $6,000, because the brother and sister were no longer interested in the property. The executor refused and told the brother that he expected the brother and sister to tender the $54,000 due on the property when the closing date arrived. The brother and sister did not do so. The brother filed suit demanding a refund of the $6,000. The executor countersued, demanding specific performance by the brother and sister or, in the alternative, monetary damages.

Absent any applicable statutes, how should the court rule?

Responses

A In favor of the executor, by requiring specific performance of the brother and sister.

B In favor of the executor, by assessing damages against the brother and sister.

C In favor of the brother, by ordering the executor to refund the earnest money.

D The court should rule that the executor is not entitled to either damages or specific performance and that the brother is not entitled to a refund of the earnest money.

A

The court should require specific performance of the brother and sister. When a transfer of land is preceded by a contract for sale, the risk of loss to the property during that time interval is imposed on the buyer in most jurisdictions. Thus, despite a loss due to fire or other casualty (assuming it was not due to the fault of either party), the buyer must still pay the contract price at the closing date unless the contract provides otherwise. Consequently, the brother and sister were still under a duty to tender the amount due on the property. Their failure to do so puts them in breach of contract, allowing the executor to obtain specific performance (payment of the balance due on the contract). (B) is incorrect because the preferred remedy for the executor is specific performance. While the executor could obtain a damage remedy based on the difference between the contract price and the market value of the land on the date of the breach, that would leave him with the task as executor of trying to resell the property. Because land is considered unique, a buyer is entitled to specific performance of the contract. The seller (the executor) can also get specific performance if the buyer is in breach.

73
Q

A small brewer leased a commercial building from a landlord for a period of five years. The brewer installed tanks, pipes, and other equipment for his brewing operations and began operations. Two years later, the landlord mortgaged the property to a bank to secure a loan. The landlord informed the bank that the building was currently leased to the brewer. The bank recorded its mortgage but the brewer was not informed of the mortgage. Six months before the lease term was up, the landlord defaulted on the loan. The bank foreclosed on the mortgage and acquired title to the property. When the brewer began removing his equipment from the building shortly before the end of the lease term, the bank sought an injunction to prevent him from doing so.

If the court denies the bank’s injunction, what will be the likely explanation?

Responses

A The bank was not a party to the lease agreement between the brewer and the landlord.

B The bank had notice of the brewer’s lease.

C The equipment was installed for the brewer’s commercial use and he did not intend for it to stay.

D The brewer had been given no notice of the mortgage.

A

The court should deny the injunction because the equipment constituted trade fixtures. Equipment installed for the purpose of trade by a tenant, even if it would otherwise constitute a fixture, is generally removable if it is not an integral part of the premises and the tenant pays for any damages caused by the removal. Absent an agreement to the contrary, these annexations are removable because it was not the intention of the tenant to make them a permanent part of the premises. Here, the brewer used the equipment for his business and had no intention to permanently improve the leased premises with his brewery equipment, so he should be permitted to remove his equipment.

74
Q

An owner conveyed her property to a buyer, who put the deed in his suitcase and took off for a five-month tour of the world. The owner, knowing that the buyer had left the country, sold the property to an investor who was not aware of the previous transaction. The investor did not record her deed. When the buyer returned from his trip, he recorded his deed. A month later, the investor conveyed the property to a developer. The developer knew that the buyer held a deed to the property but completed the transaction anyway. Instead of recording, however, the developer immediately filed an appropriate action against the buyer and against the investor to determine ownership of the property.

The property is situated in a jurisdiction containing the following statute: “Any conveyance of an interest in land, other than a lease for less than one year, shall not be valid against any subsequent purchaser for value, without notice, unless the conveyance is recorded.”

How should the court rule?

Responses

A The buyer has rights superior to those of both the investor and the developer.

B The investor has rights superior to those of both the buyer and the developer.

C The developer has rights superior to those of both the buyer and the investor.

D The developer has rights superior to those of the investor, but the buyer has rights superior to those of the developer.

A

The court will rule that the developer has rights superior to those of the other parties. Under a notice statute, which is the statute applicable here, the general rule is that a subsequent bona fide purchaser (“BFP”) will prevail over a prior grantee who failed to record before the BFP’s purchase; if the prior grantee has previously recorded, the subsequent purchaser ordinarily will be deemed to have record notice of the prior conveyance and will not be a BFP. Here, the owner resold the property to the investor before the buyer recorded his deed to the property; thus, the investor had no record notice of the transfer. Because the investor also had no actual knowledge or inquiry notice (the buyer was out of the country), she qualifies as a BFP. Finally, under the shelter rule, a person who takes from a BFP will prevail against any interest that the transferor-BFP would have prevailed against. Thus, even though the developer knew that the buyer held a deed to the property, the developer prevails against the buyer because the investor would have prevailed against the buyer.

75
Q

A landowner and her neighbor owned adjacent parcels of land. The landowner hired a contractor to install an in-ground swimming pool on her land. The day after the contractor had excavated for the pool, the neighbor’s storage shed, located on his property a few feet from the edge of the excavation, collapsed when the ground shifted. A riding tractor and patio furniture contained within the shed were damaged. The neighbor sued the landowner for damages. At trial, the neighbor established that the landowner’s project caused the subsidence and the damage to his property.

What else must the neighbor establish to prevail?

Responses

A No additional facts.

B That his land would have been damaged without the storage shed.

C That the contractor was negligent.

D That his land would have been damaged without the storage shed or that the contractor was negligent.

A

For the neighbor to prevail, he must show either that his land would have been damaged without the storage shed or that the contractor was negligent. A landowner has a right to have his land supported in its natural state by adjoining land. If, however, the land has buildings on it, an excavating adjacent landowner is strictly liable for damage to the buildings caused by the excavation only if the excavation would have caused the land to subside even in its natural state (i.e., without buildings). Even if the land would not have subsided in its natural state, the excavating landowner is liable for the damages if she was negligent. Here, the neighbor has established that the landowner’s excavation caused the damage to the neighbor’s storage shed. To prevail, the neighbor will need to show either that the subsidence would have occurred even if the land had been unimproved or that the contractor hired to excavate for the pool was negligent.

76
Q

The owner of a factory that uses widgets in making its product and a widget maker entered into negotiations over the telephone and, after a time, reached a general understanding that the factory owner would buy widgets from the widget maker. Following their conversation, the widget maker sent the factory owner a contract, which he had already signed, agreeing to sell 1,000 widgets to the factory owner for a total contract price of $10,000. Upon receipt of the contract in the mail, the factory owner signed the contract and deposited an envelope containing the contract in the mailbox located in front of his workplace.

Before the widget maker received the contract, the factory owner had a change of heart. He telephoned the widget maker and told him that he could not afford to buy the widgets he had ordered, and he was “not interested in that contract we talked about.” The widget maker replied, “That’s all right, I understand. Maybe we can do business some other time.” The next day, the signed contract was delivered to the widget maker’s office. The widget maker, also having had a change of mind, decided that he wanted to enforce the contract.

Is the contract enforceable against the factory owner?

A

The contract is enforceable because the mailbox rule applies here. Acceptance by mail creates a contract at the moment of posting, properly addressed and stamped, unless the offer stipulates that acceptance is not effective until received, or an option contract is involved. If the offeree sends an acceptance and then rejects the offer, the mailbox rule applies; i.e., a contract is created upon dispatch of the acceptance. Because no option contract is involved here, and the widget maker’s offer did not state that the factory owner’s acceptance would be effective only when received, his acceptance was effective the moment he placed the envelope containing the contract in the mailbox. His attempt to reject occurred after acceptance took place. Thus, a valid contract was formed and the widget maker may enforce it.

77
Q

After reaching an oral agreement on the terms of representation, a law firm, at its clients’ behest, instituted a class action lawsuit against a tobacco company for $100 million. Prior to signing the written contract outlining the parties’ rights and responsibilities, including the fee arrangement, the firm’s senior partner told the clients’ representative in a moment of goodwill and generosity that if they won or the tobacco company settled, he would turn over half of the attorneys’ fees in the case to a particular nonprofit group that funds research on lung cancer and other respiratory illnesses. After the law firm won the case and collected its fee of $33 million, it had second thoughts about turning over half of it to the nonprofit group.

If the nonprofit group sues the law firm in an attempt to collect the $16.5 million, which of the following is the law firm’s best defense to such action?

A

The nonprofit group did not give consideration to the law firm in return for the law firm’s promise to turn over half of its attorneys’ fees to the group in the event it won or settled the class action suit. Thus, the law firm’s promise was gratuitous; i.e., it was simply attempting to confer a gift upon the nonprofit group, and the group could not compel the law firm to turn over the money.

78
Q

On November 5, an electronics store owner realized that his stock of 15 copies of the most popular video game of the holiday shopping season would not last until the first of the next month. Seeing an advertisement from the manufacturer of the game in a trade journal listing its price at $3,000 per hundred, with delivery one week from order, the store owner e-mailed to the manufacturer an order for 100 copies of the game at $3,000 per hundred. There were no further communications between the store owner and the manufacturer. By November 25, the store owner realized that the manufacturer was not going to deliver any of the video games. He thus was forced to obtain additional stock by purchasing from a middleman at a cost of $4,000 per hundred. The store owner brings an action for breach of contract against the manufacturer.

Who will prevail?

A

The manufacturer will prevail because it never accepted the offer. For a communication to be an offer, it must contain a promise, undertaking, or commitment to enter into a contract, rather than a mere invitation to begin negotiations. The broader the communicating media, e.g., publications, the more likely it is that the courts will view the communication as merely the solicitation of an offer. An advertisement in a trade journal generally is construed as an invitation to submit offers, not an offer itself. It is an announcement of the price at which the seller is willing to receive offers. Thus, the store owner’s e-mail was an offer that was never accepted by the manufacturer.

79
Q

An officer on routine patrol noticed a flashlight moving within a darkened house and stopped to investigate. The suspect, who had broken into the home to steal valuables, caught sight of the patrol car, dropped the bag of valuables as he was about to carry them out of the house, and tried to sneak out the back way. The officer saw him sneaking out and seized him. The suspect, who had a lock-picking device in his possession, pulled out two $100 bills from his wallet, stating that he did not take anything and would like to forget the whole thing. The officer took the money, stating that she would give him a break this time around, and let the suspect go.

How may the officer be charged in this situation?

A

The officer is an accessory after the fact to burglary. An accessory after the fact is one who receives, relieves, comforts, or assists another, knowing that he has committed a felony, in order to help the felon escape arrest, trial, or conviction. The crime committed by the principal must have been completed at the time aid is rendered. Here, the officer had a duty to arrest the suspect and failed to do so, instead letting him go. Her failure to act under these circumstances constituted sufficient assistance to the suspect to make her liable as an accessory after the fact. She almost certainly knew that the suspect had committed the felony of burglary, as she saw him with a flashlight in the darkened house and caught him sneaking out the back way with a lock-picking device in his possession. Under these facts, she can be liable as an accessory after the fact to burglary.

80
Q

A breeder of quarter horses entered into an agreement with a rancher to sell and deliver two quarter horses, one to the rancher and the other to the rancher’s fiancée as a gift. Although the fair market value of each horse was $3,000, the horse breeder agreed to sell both horses together for a total price of $5,000. Under the agreement that the rancher wrote out and both parties signed, the horse breeder agreed to deliver one horse to the rancher on August 1, at which time the rancher agreed to pay the horse breeder $5,000. The horse breeder further agreed to deliver the other horse to the rancher’s fiancée on August 12.

On August 1, the horse breeder delivered the first horse to the rancher and, at the same time, the rancher gave the horse breeder a certified check for $5,000. On August 12, the horse breeder brought the second horse to the residence of the rancher’s fiancée and told her that the horse was a gift from the rancher. The rancher’s fiancée told the horse breeder that she loathed quarter horses and she refused to take the horse. The horse breeder brought this horse back to his farm and sent an e-mail to the rancher, informing him that his fiancée refused delivery and that he (the horse breeder) could not keep the horse. Two weeks later, after not hearing from the rancher, the horse breeder sold the horse to an interested party for $3,000.

If the rancher sues the horse breeder, how much should the rancher recover?

A

The rancher should recover $2,000 because that is the amount by which the horse breeder would be unjustly enriched.

81
Q

On February 1, a national department store chain entered into a written agreement with a canoe manufacturer providing that the manufacturer would sell the department store any quantity of 16-foot aluminum canoes that the department store desired at a price of $250 per canoe, deliveries to be made 30 days after any order. The agreement was signed by authorized agents of both parties. On March 1, the department store sent the manufacturer an order for 500 canoes to be delivered in 30 days. The manufacturer immediately e-mailed the department store a confirmation of the order. Ten days later, the department store sent the manufacturer an order for an additional 500 canoes, to be delivered in 30 days. Five days after receiving the department store’s second order, the manufacturer e-mailed the department store and explained that a large sporting goods chain was willing to purchase all of the manufacturer’s output of 16-foot canoes at $275 per canoe and that the manufacturer would be unable to fill any of the department store’s orders.

The department store found another canoe manufacturer willing to provide it with 16-foot aluminum canoes for $280 per canoe and on April 15 filed an action against the manufacturer seeking damages for the manufacturer’s failure to deliver the 1,000 canoes ordered.

How should the court rule?

A

This question is best answered by eliminating the incorrect choices first. (A) is incorrect. A contract was formed here for 500 canoes. The original “agreement” between the parties was nothing more than an invitation seeking offers. It did not create a contract between the department store and the manufacturer because it was illusory—an agreement to buy only what is desired is not consideration. The “agreement” probably does not even qualify as an offer. An offer must express a commitment to conclude a bargain on the offered terms. Absent some quantity limitation, a court would probably find the “agreement” here too vague to constitute an offer; otherwise, the manufacturer could be committing itself to sell more canoes than it can supply. Thus, the department store’s first order will be construed as an offer, and the manufacturer’s confirmation will be construed as an acceptance of the offer, thus creating a contract.

82
Q

A massive earthquake struck Mexico, causing widespread death and destruction. The President of Mexico made an urgent plea to the President of the United States asking for assistance. Congress was not in session and the President, without calling Congress into special session, ordered extensive military aid, in the form of personnel and equipment, to assist Mexico.

Which of the following best describes the constitutionality of the President’s orders?

Responses

A The action was a valid exercise of the President’s position as Commander in Chief of the Armed Forces of the United States.

B The action was valid under the plenary powers of the President, inasmuch as the Army units were used for humanitarian, rather than warlike, purposes.

C The action was unconstitutional, because the President may not perform acts when Congress is not in session that he would have to ask Congress to approve if it were in session.

D The action was unconstitutional, because the President does not have the power to unilaterally authorize this type of foreign aid.

A

The President as Commander in Chief of the Armed Forces has broad discretion to use American troops in foreign countries. In fact, never in all of American history has a presidential use of troops in a foreign nation been declared unconstitutional. Thus, the President’s power as Commander in Chief would justify this action.

83
Q

The President issued an executive order prohibiting exportation to certain countries of specific computer software that, although not usable directly to develop nuclear weapons, would facilitate nuclear weapons technology. Congress had previously passed a law authorizing the issuance of such orders. Prior to the issuance of the executive order, a computer software company contracted with one of those countries for software that is now banned for sale and distribution.

What effect does the executive order have on this contract?

Responses

A The executive order unconstitutionally impairs the obligation of the company’s preexisting contract, which was lawful when made.

B The executive order unconstitutionally denies the company a valuable property interest without due process, because it is not limited to computer software used directly to produce nuclear weapons and, therefore, is not necessary to vindicate a compelling national need.

C The executive order is constitutional because Congress has plenary powers to regulate commerce with foreign nations and has used that power to authorize such orders.

D The executive order is constitutional, because the inherent power of the President to conduct foreign affairs is plenary.

A

The executive order is constitutional because it was authorized by Congress. For all practical purposes, the power to regulate foreign commerce lies exclusively with Congress, and Congress may delegate this power to the President. By authorizing the issuance of the Presidential executive order, Congress has constitutionally exercised this power.

84
Q

The state capitol building had fallen into disrepair, so the state contracted with a company for remodeling. After the contract was signed but before remodeling work had started, the state repealed the statute authorizing money for remodeling and notified the company that it was cancelling the contract because of budgetary concerns.

If the company brings suit in federal court against an appropriate state official to enjoin cancellation of the contract, for which party should the court rule?

Responses

A For the company, because it had a valid contract.

B For the company, if it has detrimentally relied on its contract with the state.

C For the state official, because the courts will not substitute their judgment for that of the legislature.

D For the state official, because constitutionally the sovereign is not liable except with its own consent.

A

The court should hold for the company because it has a valid contract. A state violates the prohibition against impairing the obligation of contract when it substantially impairs any contracts to which it is a party and the impairment does not meet the various exceptions recognized by the Court. Here, the state is simply seeking to reduce its contractual financial obligations, and its decision would violate the prohibition.

85
Q

A United States senator made a speech on the Senate floor asserting that the President has a swastika tattoo in a location usually covered by clothing. The assertion is demonstrably false, and the senator knew that it was either false or very unlikely to be true. The President has sued the senator for defamation.

Which of the following is the best reason for the court to dismiss the complaint?

Responses

A The President’s tattoos are not a matter of public concern.

B The statement was not made with actual malice.

C The complaint presents a nonjusticiable political question.

D The statement was made by a senator on the Senate floor.

A

The best reason for the court to dismiss the complaint is that the senator’s statement was made on the Senate floor and is therefore protected by the Speech or Debate Clause. Article I, Section 6 provides that “For any speech or debate in either House [members of Congress] shall not be questioned in any other place.” Senators who make statements on the Senate floor have immunity from defamation suits, even if the defamation suit could otherwise overcome the high standard for suits against public figures.

86
Q

A defendant stood accused of murdering a family of six in a small town. The judge, concerned not only about prejudice to the defendant’s right to a fair trial but also about media attention and public opinion in such a small locale, issued an order forbidding the press from attending the trial or publishing any details of the testimony at trial.

If a local newspaper sues in federal court to have the judge’s gag order overturned, will the newspaper prevail?

Responses

A Yes, because the proper way to deal with prejudicial publicity is via a change of venue.

B Yes, because the newspaper has a Sixth Amendment right to a public trial.

C Yes, because the judge has attempted to impose a prior restraint in violation of the First Amendment.

D No, because the judge honestly believed that publicity would be prejudicial and would impair the defendant’s constitutional right to a fair trial.

A

The newspaper will prevail on First Amendment grounds. In Nebraska Press Association v. Stewart (1976), the Supreme Court made it clear that a prior restraint will be upheld only if it is the only sure way of preserving a fair trial for the defendant. Although prejudicial publicity can pose a serious problem, the Supreme Court held that it virtually always should be dealt with by means other than prior restraints.

87
Q

A controversial religious group had several long-standing disputes with local politicians. Recently, the county fire marshal inspected the church’s building and found it failed to meet code because of insufficient fire exits. As was standard procedure for all such violations, the fire marshal initiated proceedings to revoke the church’s certificate of occupancy. The group requested that the county make an exemption from the building code, asserting that it could not afford to make the necessary changes to make the building conform to the code, and that closing the building would hinder their ability to congregate and worship.

If the group’s request is denied and the group seeks to enjoin the county from enforcing the building code on constitutional grounds, which party is likely to prevail?

Responses

A The church on the basis of equal protection.

B The church, on the basis of freedom of religion.

C The county, because an exemption for the church would constitute an establishment of religion.

D The county, because the regulation was not intended to close the church.

A

The county should prevail because the regulation was not intended to close the church and it appears that it was not applied in a discriminatory manner. A state regulation promulgated to protect the health and welfare of the citizens of the state will prevail over constitutional objections, as long as the state regulation was not instituted merely to discriminate against a particular religious group and was not applied in a discriminatory manner. In this case, the building code was enacted to protect the health and safety of the state’s citizens and not simply to close down the church, and it was standard procedure to revoke occupancy certificates for the type of violation found.

88
Q

A bottler markets water in lightweight plastic bottles that are sold by grocery stores, sporting goods stores, and other retail outlets. A hiker purchased several bottles of the water from a retailer and took them with him on a hike. While the hiker left his backpack unattended, a thief took one of the unopened containers without permission and drank some of the water. He immediately became violently ill. Tests were run on the water and showed that it contained impurities.

If the thief maintains a negligence action against the bottler, which of the following arguments would be the most helpful to the bottler in avoiding liability?

Responses

A The retailer had ample opportunity to test and inspect samples of the bottled water for purity and failed to do so.

B The bottler bottled its water in compliance with numerous statutes that regulate the process of bottling water for human consumption.

C The thief has failed to introduce any evidence at trial as to how the impurities got into the water he drank, and therefore has not met his burden of proof.

D No reasonable person would have foreseen that the water would have been stolen and consumed by a thief.

A

Evidence that the bottler complied with applicable statutes will be admissible to show that the bottler acted with ordinary, reasonable care, and is the only one of the listed arguments that would be helpful to the bottler. The bottler is being sued on a negligence theory; thus, the thief must prove that the bottler failed to exercise ordinary, reasonable care in bottling and distributing the water. Violation of a statute will establish a conclusive presumption of duty and breach of duty. However, compliance with an applicable statute does not necessarily establish due care, because due care may require more than is called for by the statute. Nevertheless, compliance with a statute is admissible as evidence that a defendant may have acted with due care. Thus, the bottler could use its compliance with the water bottling statutes as a means of establishing that it conformed with its duty to use ordinary, reasonable care.

89
Q

A fire broke out in a home that had been recently remodeled, destroying the house and injuring the homeowner. An investigation by the fire marshal established that the fire started from a short in some wiring behind a wall. A small section of wiring that ran to an outlet through a narrow gap between a furnace chimney and a hot water pipe had part of its outer sheath cut off. The homeowner filed suit against the electrical company that did the rough wiring.

The parties stipulated for trial that the company had installed the wiring in compliance with the blueprints, and that the wiring had been inspected and approved by the building inspector before the chimney and the water pipe had been installed and the walls put up, all by different contractors. At trial, the homeowner introduced the report of the fire marshal establishing how the fire started, and evidence of his medical expenses and other damages. At the end of the homeowner’s case, the electrical company’s attorney rested her case and moved for a directed verdict. The homeowner’s attorney also moved for a directed verdict.

How should the court rule on the directed verdict motions?

A

The court should grant the electrical company’s motion for a directed verdict because the homeowner has not established a prima facie case of negligence on the company’s part. The homeowner has established that the electrical company owed a duty to him and that he has suffered harm from the fire caused by the short in the wiring. However, he has not established that the company breached any duty to him. While breach of duty is ordinarily a question for the trier of fact, a plaintiff’s failure to offer any evidence on that element of the prima facie case will permit a directed verdict for the defendant.

90
Q

Two neighbors who worked in a large city nearby alternated days driving. Because the commute took them through a crime-ridden area, one commuter was vigilant about keeping her car well-maintained, but the other failed to maintain her car or bring it in for servicing, despite the first commuter’s complaints and dashboard warning lights indicating that it needed servicing. One evening after dark when the latter was driving them both home from work, her car died just as they were passing through a dangerous neighborhood. The passenger, who was calling for assistance on her cell phone, protested when the driver opened her door to look at the engine. Two assailants appeared and beat and robbed the driver and passenger.

Does the passenger have a valid claim against the driver for her injuries?

A

The passenger has a valid claim against the driver because the jury could reasonably conclude that she was negligent. The driver owes a duty of ordinary care to his passenger regardless of whether that passenger is paying or not paying. The driver also owed a duty to act as a reasonable person would under emergency circumstances after the car was stopped. Her opening of the car door, as well as her failure to maintain her car, could be found to be negligent under the circumstances. The acts of the criminals were foreseeable, because every day the passenger and the driver commuted through this dangerous neighborhood.

91
Q

A passenger suffered a broken arm from an automobile accident caused by his driver’s negligence in running through a red light. The passenger was taken by ambulance to a nearby hospital for treatment. There, the emergency room physician negligently reset the bone in the passenger’s arm. As a result, the passenger never recovered full use of his arm and his earnings as a carpenter were permanently reduced. The jurisdiction retains traditional contribution rules based on equal shares in cases applying joint and several liability.

If the passenger brings suit against the driver for the damage to his arm, the passenger will recover:

A

The passenger will recover all of his damages from the driver. Because the damage resulting from medical malpractice was a foreseeable result of the driver’s negligence, the driver is liable for such damage even though the negligent doctor contributed to the passenger’s damages; i.e., the doctor’s negligence is not a superseding cause and therefore does not cut off the driver’s liability for the passenger’s injuries.

92
Q

Question
A state child safety statute required children under eight years of age to be in a government-approved car seat when riding in a motor vehicle. A father was driving to a ballgame with his seven-year-old child, who was buckled in the back seat with a regular seat belt. The father did not notice when the child unbuckled himself and started climbing into the front seat. The child grabbed the steering wheel “to help daddy steer,” causing the car to swerve into the other lane and collide with another motorist’s car. The motorist was seriously injured from the collision; the father and his child were unhurt.

The motorist sued the father to recover damages for her injuries. At trial, the motorist presented evidence of the statute, her injuries, and the facts stated above. At the conclusion of the proofs, both parties moved for a directed verdict.

How should the trial judge proceed?

A

The court should deny both motions and submit the case to the jury, because the jury could find that the father was negligent in not preventing his child from grabbing the steering wheel. Under ordinary negligence principles, the father owed a duty to other motorists to maintain control of his vehicle. The jury could find that the father breached this duty of care by not noticing when his child unbuckled the seat belt and started climbing into the front seat, and not preventing the child from grabbing the steering wheel. Hence, the case should go to the jury for a determination of whether the father was negligent.

93
Q

A landowner leased 150 acres of farmland to a produce company for 15 years. The produce company used the land for crops along with several other contiguous acres that it owned or leased. About four years into the lease, the state condemned a portion of the leased property because it intended to build a highway. As a result, too little property remained for the produce company to profitably farm, although there still existed the farmhouse on the property, which was being used by one of its foremen. The produce company gave the landowner 30 days’ written notice that it considered the lease to have been terminated because of the condemnation.

In a suit for breach of contract, is the landowner likely to win?

Responses

A No, because the condemnation made it economically undesirable for the produce company to continue to lease the property.

B No, because when there is a condemnation, the tenant’s obligation to pay rent is extinguished.

C Yes, because the produce company can still use the farmhouse, and the rental value would be adjusted accordingly.
D Yes, because the condemnation did not affect the produce company’s obligation to pay the full rental price, although it is entitled to share in the condemnation award.

A

The landowner probably will win in a breach of contract suit. In partial condemnation cases, the landlord-tenant relationship continues, as does the tenant’s obligation to pay the entire rent for the remaining period of the lease. The tenant is, however, entitled to share in the condemnation award to the extent that the condemnation affected the tenant’s rights under the lease.

94
Q

A homeowner sued a plumber for breach of contract. At trial, the homeowner intends to testify about statements the plumber made to him during a conversation in the homeowner’s kitchen. This conversation had been recorded on video by the homeowner’s “nanny cam,” but the recording is unavailable due to a technical failure outside of the homeowner’s control.

Is the homeowner’s testimony admissible?

A

The homeowner’s testimony is admissible because the homeowner has personal knowledge of what the plumber said during their conversation—the homeowner was present and heard the plumber’s statements himself.

95
Q

A backgammon player was upset after losing a match against the club champion. Rushing out of the club, he inadvertently grabbed the champion’s board, which looked very much like his own but which was much more expensive. The player left the backgammon board in the trunk of his car, as was his usual practice. During the night, the car was stolen and along with it, the champion’s expensive backgammon board.

In an action by the champion against the player to recover the value of the backgammon board, is the champion likely to recover?

Responses

A Yes, because when the player took the backgammon board he committed a trespass to the champion’s chattel.

B Yes, because when the backgammon board was stolen along with the car, the player became liable for conversion of the champion’s chattel.

C No, because the player believed in good faith that the board was his when he took it from the backgammon club.

D No, because the backgammon board was lost through no fault of the player’s.

A

The champion will recover from the player for conversion. Conversion consists of (i) an act by defendant interfering with plaintiff’s right of possession in the chattel, (ii) intent to perform the act bringing about the interference with plaintiff’s right of possession, (iii) causation, and (iv) damages—an interference that is serious enough in nature or consequence to warrant that the defendant pay the full value of the chattel. Intent to trespass is not required; intent to do the act of interference with the chattel is sufficient for liability. Therefore, the player was guilty of conversion when he intentionally (i.e., volitionally) took the champion’s board, which resulted in its loss, even though the player did not intend to lose it or even realize that he had taken the property of another.